Está en la página 1de 57

...

--~r-===;:=:;;:,





L.!.
. _...:;_;,;...____.;..___












1



1

1
1
Manual CTO
de Medicina y Ciruga

2. a edicin

Urologa

Revisores
Karla Leonor Sierra Guerra
Pablo Conde Caturla

Autores
Sara Daz Naranjo
Lai n Hermes Gonzlez Quarante
(traduccin casos clnicos)

Grupo CTO
Editorial
NOTA

La medicina es una ciencia sometida a un cambio constante. A medida que la investigacin y la experiencia
clnica amplan nuestros conocimientos, son necesarios cambios en los tratamientos y la farmacoterapia.
Los editores de esta obra han contrastado sus resultados con fuentes consideradas de confianza,
en un esfuerzo por proporcionar informacin completa y general, de acuerdo con los criterios aceptados
en el momento de la publicacin. Sin embargo, debido a la posibilidad de que existan errores humanos
o se produzcan cambios en las ciencias mdicas, ni los editores ni cualquier otra fuente implicada
en la preparacin o la publicacin de esta obra garantizan que la informacin contenida en la misma sea
exacta y completa en todos los aspectos, ni son responsables de los errores u omisiones ni de los resultados
derivados del empleo de dicha informacin. Por ello, se recomienda a los lectores que contrasten dicha
informacin con otras fuentes. Por ejemplo, y en particular, se aconseja revisar el prospecto informativo
que acompaa a cada medicamento que deseen administrar, para asegurarse de que la informacin
contenida en este libro es correcta y de que no se han producido modificaciones en la dosis recomendada
o en las contraindicaciones para la administracin. Esta recomendacin resulta de particular importancia
en relacin con frmacos nuevos o de uso poco frecuente. Los lectores tambin deben consultar
a su propio laboratorio para conocer los valores normales.

No est permitida la reproduccin total o parcial de este libro, su tratamiento informtico, la transmisin
de ningn otro formato o por cualquier medio, ya sea electrnico, mecnico, por fotocopia, por registro
y otros medios, sin el permiso previo de los titulares del copyright.

CTO EDITORIAL, S.L. 2016

Diseo y maquetacin: CTO Editorial

C/ Francisco Silvela, 106; 28002 Madrid


Tfno.: (0034) 91 782 43 30- Fax: (0034) 91 782 43 43
E-mail: ctoeditorial@ctomedicina.com
Pgina Web: www.grupocto.es

ISBN Urologa: 978-84-16S27-38-0


ISBN Obra completa: 978-84-16527-11-3
Depsito legal: M-26880-2015
Urologa

Manual CTO
de Medicina y Ciruga

2.a edicin

Grupo CTO
Editorial
Q)
(.)
-
-o
e
,_ _
. C'C
04. Tumores renales ......
4.1.

4.2.
Carcinoma de clulas renales
(adenocarcinoma renal, hipernefroma) ............................ 17
Otros tumores ........................................................................................................................ 19
17

05. Hiperplasia y carcinoma prosttico 21


5.1. Hiperplasia prosttica ben igna ...................................................... 2 1
5.2. Carcinoma prosttico ............................................................. ... . 23
01. Semiologa urolgica
y definiciones .. .. uu 1
1.1 . Definiciones ..... 06. Carcinomas del tracto urinario 29
1.2. Diagnstico diferencial 6.1. Carcinoma ves ical .............................................................................. .... . 29
de la hematuria macroscpica ..... 2 6.2. Tumores del t racto urinario superior .................................... 31

02. Infecciones del tracto urinario. 07. Tumores testiculares ... u . 34


Cistitis intersticiaL........ 3
7.1. Etiologa y epidemiologa ............................................................................ 34
2.1. Patognesis y etiologa ..... 3 7.2. An atoma patolgica ............................................................................................... 34
2.2. Diagnstico ..... 4 7.3. Clnica....................... .......................................................................................... 35
2.3. Diferentes ITU y su tratamiento ......... 4 7.4. Diagnstico .......................................... .. m 35
2.4. Tuberculosis genitourinaria .. ......... 6 7.5. Diagnstico diferencial ....................................................................................... 36
2.5. Cistitis intersticial... 7 7.6. Tratamiento ... ........................................................................................................................... 37

03. Urolitiasis 10 08. Trasplante renaL . 40


3.1. Epidemiolog a.... . .... .. 10 8.1. Indicaciones ................................................................................................................................ 40
3.2. Manifestaciones clnicas y su manejo agudo ........ 10 8.2. Cont ra indicaciones .................................................................................................... 40
3.3. Evaluacin y tratamiento de la litiasis renal ............. 12 8.3. Complicaciones ................................................................................................................... 40
,
Urologa 1 1n d i C e

09. Uropata obstructiva ... 42 11. Traumatismos del aparato


9.1 . Caractersticas .................................................................................................... 42 genitourinario .. H 46
9.2. Clnica ....................................................................................................................................................... 42 11 .1. Lesiones del rin .......................................................... . .. .. . .... 46
9.3. Diagnstico .................................................................................................................................. 42 11 .2. Lesiones del urter ....................................................................................................... 47
9.4. Tratamiento ................................................................................................................................. 43 11 .3. Lesiones de la vejiga . . . 47
11.4. Lesiones de la uretra ........................... ................. 47
11.5. Lesiones del pene . . . ............ 48
1O. Disfuncin erctil... 44 11.6. Lesiones de los testculos ........... ... . .... 48
10.1 . Introduccin ................................................................................................................ 44
10.2. Prevalencia .................................................................................................................................. 44
10.3. Etiologa ............................................................................................................................................. 44 Bibliografa ... . 51
10.4. Factores de riesgo ... .......................... 44
10.5. Diagnstico .................................................................................................................................. 44
10.6. Tratamiento ........................................................................................................................... 44
_lj[nlo:-1-"-''""'-----~

Semiologa urolgica y definiciones

Este tema puede ayudar tes quirrgicos previos) y la segunda, el urter ectpico (que es
a obtener una visin general la causa ms frecuente en nias).
de la materia y a asociar algunos De esfuerzo: se desencadena con el aumento de presin abdo-
hallazgos a patologas concretas, minal (al rer, toser, cargar con peso). Generalmente se produce
pero no se debe emplear por un dficit de soporte de la musculatura perineal (por ello es
en l demasiado tiempo.
recomendable revisa r los antecedentes obsttricos, pacientes
obesas, pacientes aosas, et ctera).
De urgencia: el paciente siente ganas de orina r, pero no le da
1.1. Definiciones tiempo a llegar al bao (provocadas por contracciones involun-
ta rias del mscu lo detrusor).
Mixta : generalmente es una combinacin de las dos anteriores.
Hematuria microscpica: presencia de ms de 5 hemates por Paradj ica: escape de orina debido a la sobredistensin vesi-
campo. La causa ms frecuente en ambos sexos es la litiasis. La causa ca l. El ejemplo caracterstico es el paciente prosttico con re-
mas comn en varones mayores de 50 aos es la hiperplasia benig- tencin urinaria. La presin intravesical supera la presin de
na de prstata. cierre del esfnte r uretral, producindose un escape de orina
Hematuria macroscpica : orina de aspecto roj izo a simple vista de- paradj ico (no puede orinar y, sin embargo, se le escapa la
bido a la presencia de ms de 50 hemates por campo. En los pacien- orina).
tes fumado res, en ausencia de otros sntomas, se debe sospechar
tumor urotelial.
Piuria: presencia de ms de 1O leucocitos por campo. Altamente
Los cil ind ros hemticos aparecen en las glomerul o-
inespecfica, pero en presencia de sntomas urinarios, hay que sos- nefritis que producen sndrome nefrtico, como en la
pechar infeccin. posti n fecc i osa.
Sndrome miccional: presencia de polaqu iuria (aumento en la fre-
cuencia miccional), urgencia miccional (necesidad imperiosa e irre-
frenable de orinar) y d isuria (molestias urinarias inespecficas referi- Enuresis: prdidas de orina exclusivamente durante el sueo. Si el
das como ardor, escozor, etctera). nio es mayor de 6 aos, debe ser estudiado.
Incont inencia urinaria: prdidas involuntarias de orina (Tabla 1}. Cri sis renoureteral: dolor lumbar frecuentemente irrad iado a ge-
Existen cuatro tipos principales: nitales, de carcter agudo, cuya intensidad no se modifica por los
Continua: de da y de noche, en todas las posiciones. La causa cambios postura les, y que se suele acompaar de nuseas, vmitos
ms frecuente es la fstula urinaria (en pacientes con anteceden- y malestar genera l. Es muy poco frecuente que sea bilateral.

Sntomas Incontinencia de urgencia , Incontinencia de esfuerzo

Urgencia (deseo repentino de orinar) S No


Aumento de la frecuencia miccional S No
Capacidad de llegar al bao despus de sentir el deseo de orinar No S

Despertarse para ir al bao durante la noche S Genera lmente no

Escape durante la actividad fsica No S

Cantidad de orina escapada en cada episodio de incontinencia Abundante, si se produce Generalmente escasa

Tabla 1. Diagnsti co diferencial de la clnica de incontinencia urinaria femen ina


Manual CTO de Medicina y Ciruga, 2.a edicin

1.2. Diagnstico diferencial Hematuria con cogulos: indica un problema urolgico. La causa
ms frecuente en mujeres es la cistitis hemorrgica, aunque la pri-
de la hematuria macroscpica mera causa a descartar es una neoplasia urotelial, mxime en el pa-
ciente fumador.
Segn el momento de aparicin: Hematuria por nefropata mdica: no suele tener cogulos, y
Inicial: sangrado uretral o prosttico. puede ir acompaada de cierto grado de proteinuria, as como de
Final: sangrado del cuello vesical. ci lindros eritrocitarios o de hemates dismrficos en el sedimento
Total: vesical o del tracto urinario alto. Se debe recordar que un urinario.
sangrado importante de cualquier parte del aparato genitouri-
nario puede provocar hematuria total.

" La hematuria con cogulos indica un problema urolgico.


Ideas clave ~
" La causa ms habitual de hematuria es la cistitis hemorrgica,
" La causa ms frecuente de hematuria microscpica es la litiasis pero lo primero a descartar es el tumor urotelial.
(en la poblacin general, en ambos sexos).
" Los hemates dismrficos en el sedimento orientan a nefropata
" La causa ms comn de hematuria microscpica en varones de de origen glomerular.
ms de 50 aos es la hiperplasia benigna de prstata.

eaS e St udy .: A 63-year-old woman presents to the emergency department


complaining of frequent urine leakage. She says these leaks are
especially notable when coughing, laughing or sneezing. She al so
A 7-year-old boy is brought to our clinic with a history of noctu- claims that sometimes she has no time to reach the bathroom.
ria, five or six times a week. Regarding this clinical case, which of Which of the following options is the most correct management?
the following da uses is false?
1) Symptoms are consistent with emergency urinary incontinence
1) lt should be considered monosymptomatic nocturnal enuresis and should be treated with anticholinergic agents.
if the patient does not ha ve daytime symptoms. 2) Symptoms are consistent with emergency urinary incontinence
2) The usual clinical course is to tend to spontaneous resolution, and the patient should initially perform pelvic floor exercises.
as time goes by. Should these exercises not be effective, a suburethral mesh pla-
3) Usually, these kids continue to present problems to control the cement procedure is indicated.
urinary sphincter when they reach adulthood. 3) Mixed urinary incontinence should be suspected, and indepen-
4) In 60% of cases, family history of similar symptoms may be en- dent treatments for both etiologies would be beneficia l.
countered. 4) In this particular case, pelvic floor muscle training will not be
effective.
Correct answer: 3
Correct answer: 3

01 Semiologa urolgica y definiciones


Urologa_______ ___


Infeccion es del tracto ur1nar1o.
Cistitis interstici al

ORIENTACIN Este tema es el ms importante de esta asignatura. No se debe bajar la guardia


con la tuberculosis genitourinaria ni en la cistitis intersticial. Hay que formar una imagen mental
ENARM tpica para reconocerlas en el caso clnico.

La infeccin del tracto urin ario (!TU) puede clas ificarse de varias fo rmas. Otro dato que apoya la importa ncia de la va ascendente es la frecuencia
Se puede hacer una divisi n anatm ica entre las \TU altas (infecciones de infeccin tras el cateterismo uretral, que es del 1% en los pacientes
renales) y las \TU bajas (cistouretri t is, prostatitis). As im ism o, la clasifica- ambulantes, y en tres o cuatro das alcanza a casi la totalidad de los pa-
cin puede basa rse en la asociacin o no de complicaciones. Una \TU cientes sondados con sistemas de drenaje abiertos. En los pacientes hos-
no complicada es un cuad ro clnico caracterizado por la presencia de pita lizados, el riesgo de infeccin alcanza un 5% por cada da de sondaje,
escozor micciona l, urgenc ia y frecue ncia, acompaado o no por hema- incluso con sistemas cerrados.
turia terminal, dolor hipogstrico, y ms raramente, febrcula . Dentro de
este grupo se podran incluir las pielonefritis no complicadas, que se Una vez que las bacterias han alcanzado el tracto urinario, tres factores
presentan como cuadros febriles con hipersensibilidad en fosa lumbar, determinan el desarrollo de la infeccin:
fiebre, nuseas o vm itos, y si n los facto res que convierten la \TU en La viru lencia del microorgan ismo.
"complicada': como son: prese ncia de cat teres, uropata obstructiva, El tamao del inculo.
refl ujo vesicoureteral, anom alas anatm icas, insuficiencia renal o tras- Los mecanismos de defensa del husped .
plante rena l. La !TU en el varn debe considera rse esencialmente "com-
plicada" de entrada. La mayora de las infecciones en la comunidad estn producidas por gr-
menes gramnegativos, principa lment e E. coli, responsab le del 85% y, en
La reapa ri cin de una infeccin t ras el trata mi ento puede debe rse a menor proporcin, Proteus, Klebsiel/a o Pseudomonas. Entre los gra mposi-
reinfecc in o recid iva. El primer trm ino expresa la infeccin nueva por tivos, nicamente el Staphylococcus saprophyticus t iene relevancia, pro-
un germen distinto al inicia l, mientras que recidi va indica infeccin por duciendo e\1 0-15% de las \TU en mujeres jve nes (segundo germen ms
el mismo germen. Esta ltima es mucho ms infrecuente que la rein- frecuente en esta poblacin).
feccin y puede estar ocasionada por litiasis infectiva, prostatitis cr-
nica, fstulas vag inales o intestinales, divertcul os vesicales infectados, Alrededor del 30% de las mujeres con clnica miccional presentan re-
cuerpos extraos, necrosis papilar infectada y otras causas que generan cuentos menores de 1oscolonias por mili litro (1 osUFC/ml); de estas, tres
un reservorio de microorganismos que d ifcilmente se eliminan con el cuartas partes presentan piuria; en el resto, existen pocos datos que de-
antibitico. muestren infeccin, y en general se tratan segn la clnica. En la orina de
las pacientes sintomticas con piuria se pueden encontrar (considern-
dose infecc in activa) rec uentos ms bajos (102-104) de los patgenos
2.1. Patognesis y etiologa habitua les. En otras ocasiones, el cuadro se justifica por la presencia de
uretritis causada por N. gonorrhoeae o C. trachomatis. El papel patgeno
de grmenes como U. urealyticum o Mycoplasma hominis est mal defini-
Existen tres posibles vas por las que los m icroorgan ismos pueden alean- do, ya que se desconoce su potencia l como uropatgenos aislados.
za r el tracto urinario: hemat gena, linft ica y ascendente. La va linftica
ca rece de importancia rea l. La diseminacin hematgena tampoco es En las infecciones nosocomiales, los grmenes gramnegativos continan
frecuente. La ms comn es la ascendente iniciada en la uretra. Probable- siendo los ms frecuentes. Si bien E. coli es el ms habitual, su frecuencia
mente por esta razn es mucho ms habitual la \TU en mujeres, dado que desciende hasta el 50% y adqu ieren mayor importancia Proteus, Klebsiella,
su uretra es muy corta y ancha, y por ello favorece el paso de microorga- Pseudomonas, Enterobactery Serratia. El 25% restante est ocasionado por
nismos hacia nive les ms altos del TGU. grmenes grampositivos como estreptococos y estafilococos. Candida
Manual CTO de Medicina y Ciruga, 2. a edicin

a/bicans puede aparecer principalmente en pacientes diabticos, catete- En el caso de los pacientes sondados permanentemente, la presencia de
rizados o con tratamientos antibiticos prolongados. bacteriuria asintomtica no es una indicacin de tratamiento y, actualmen-
te, incluso es dudosa la recomendacin clsica de empleo profilctico de
La afectacin del tracto urinario superior parece tambin producirse por as- algn antibitico, previo a la sustitucin del catter, a fin de contrarrestar la
censo de los grmenes a lo largo del urter. La diferenciacin, aunque poco posible diseminacin hematgena del germen producida por la manipu-
especfica, se debe basar en los hal lazgos clnicos (fiebre, dolor lumbar, esca- lacin (las ltimas guas clnicas ya no lo recomiendan). S es, sin embargo,
lofros) y analtica elemental (leucocitosis, velocidad de sed imentacin alta). indicacin de tratamiento la bacteriuria persistente a los 3-5 das de haber
retirado una sonda vesica l. En aquellos pacientes en los que la sonda no
pueda ser retirada, el tratamiento de las bacteriurias asintomticas no suele
2.2. Diagnstico ser efectivo, y puede dar lugar a seleccin de cepas resistentes. En estos
pacientes slo se debe iniciar tratamiento si presentan alto riesgo de desa-
rrollar bacteriemia o si la bacteriuria se hace sintomtica.
El diagnstico de ITU, adems de la clnica, se define por el cu ltivo de
orina. Dado que es frecuente el crecimiento de bacterias que han con- En el resto de los casos, nicamente con la concurrencia de factores par-
taminado las muestras, se utiliza un criterio estadstico sobre la base del ticulares, se debe tratar la bacteriuria, y siempre sobre la base del estudio
recuento de colonias del urocultivo, considerando como significativo cl- de sensibilidades.
sicamente el crecimiento de ms de 1oscolonias por mililitro.
ITU baja en mujeres
En determ inadas circunstancia s, recuentos de colon ias menores pueden
ser suficientes: recuentos de 103 UFC/ml en mujeres sintomticas, ms de Puede realizarse un tratamiento convencional de siete das o bien un cur-
104 en pielonefritis clnicas o en varones, y ms de 102 en muestras de ca- so corto en monodosis o en rgimen de tres das. La ventaja de estos es
teterismos limpios o cualquier recuento, si se recoge mediante puncin- el menor coste econm ico y la menor incidencia de efectos adversos. Su
aspiracin suprapbica. Cifras mayores de 1osUFC/ml pueden igualmen- desventaja es la mayor incidencia de recurrencias tempranas, al no afec-
te refleja r contaminacin, principalmente si crecen dos o ms especies. tar apenas a los reservorios vag inal e intestinal de uropatgenos. Aun con
todo, por las ventajas mencionadas, la pauta preferida actualmente es el
En el adulto, la presencia de piuria (ms de 1Oleucocitos/mm 3) se relacio- tratamiento de tres das.
na estrechamente con la ITU en presencia de sntomas, no as en el nio,
en el que puede acompaar a los cuadros febriles. Los antibiticos de eleccin deben ser la nitrofurantona y la fosfomici-
na, seguidas del cotrimoxazol. Las fluoroquinolonas, las cefalosporinas y
otros P-lactmicos deben reserva rse para los cuadros complicados, con
2.3. Diferentes ITU y su tratamiento ma la respuesta previa o graves.

En mujeres embarazadas se recomiendan las pautas largas de tratamien-


En el tratamiento de la ITU lg icamente es fundamental el empleo de to (siete das), evitando el uso de su lfamidas al final del embarazo por
antimicrobianos. El nmero empleado de estos es elevado y las pautas el ri esgo incrementado de kernicterus, y el empleo de quinolonas por el
de tratamiento muy variables. A continuacin, se repasarn las opciones dao prod ucido sobre el ca rtl ago de crecimiento fetal. Tampoco se em-
teraputicas segn el tipo de ITU a la que uno se enfrente. plearn pautas cortas en caso de sospecha de pielonefritis, presencia de
clculos o anomalas de la va urinaria, o bien infecciones previas por mi-
Bacteriuria asintomtica croorganismos resistentes a los antibiticos.

Defin ida como bacteriuria significativa (1 osUFC/ml) en, al menos, dos uro- ITU recurrente
cultivos con el mismo germen, tomados con una semana de diferencia en
ausencia de sntomas. La bacteriuria asintomtica no debe tratarse salvo en Aparicin de cuatro o ms episodios al ao. Se puede realizar profilaxis
los casos en los que conlleva un riesgo de infeccin cl nica o dao orgn i- con cotrimoxazol o una fluoroquinolona (en funcin de la sensibilidad
co, como ocurre en nios menores de S aos, tengan o no patologa urol- del germen aislado en el ltimo episod io) en dosis nica, das alternos,
gica asociada. Asim ismo, debe ser tratada en el embarazo, en pacientes in- durante seis meses. Si tras la retirada se presentaran nuevas recurrencias,
munodeprimidos, como profilaxis previa a una ciruga urolgica (Tabla 2). puede reinstaurarse el tratamiento durante periodos ms prolongados
(1-2 aos). Es aconsejable la ingesta abundante de agua y realizar miccio-
nes frecuentes y cumpl ir una serie de reglas bsicas higinico-dietticas.
Menores de 5 aos
Embarazadas Si los episodios tienen relacin con el coito, se puede administrar un
lnmunodeprimidos comprimido de cotri moxazol o una quinolona despus del mismo. En
Previamente a ciruga urolgica mujeres posmenopusicas, el tratamiento con estrgenos tpicos vagi-
Tabla 2. Bacteriuria asintomtica: indicaciones de tratamiento nales disminuye la frecuencia de infecciones.

Proteus es intrnseca mente resistente a las nitrofuran-


tonas, ya que alcaliniza la ori na gracias a su ureasa, Staphylococcus saprophyticus se ha relacionado con
y este grupo de antibiticos nicamente es til en me- ITU en mujeres jvenes sexua l mente activas.
dio c ido. Recuerda

02 Infecciones del tracto urinario.


Cistitis intersticial
Urologa 1 02
Pielonefritis aguda no complicada
En pacientes con SIDA, Cryptococcus neoformans
puede ser una causa de prostatitis, ya que se elimina
En los casos de gravedad leve-moderada, puede plantea rse teraputica a travs de la o rina.
oral con cotrimoxazol (en desuso en nuestro medio por el elevado ndice
de res istencias), fluoroqui nolonas o P-lactmicos. En pacientes graves u
hospita lizados es preciso tratamiento parenteral, y el espectro de antimi- La prostatitis crnica bacteriana suele presentarse como mo lestias
crobianos incluye ampicilina (enterococo), ureidopenicilinas (Pseudomo- perineales o genita les, sntomas irritativos (polaquiuria, tenesmo, es-
nas), cefalosporinas de segunda o tercera generacin, e incluso amino- cozor) y episodios de ITU recurrentes causados por el mismo orga-
glucsidos. Nunca se emplearn pautas cortas. nismo. En el lquido prosttico se evidencian ms de 1O leucocitos
por campo de gran aumento, y macrfagos que contienen cuerpos
El antibitico empleado se seleccionar, por supuesto, sobre la base del ovales grasos.
cu ltivo y del antibiograma, y cuando se inicie de forma emprica, habr
que tener en cuenta factores que orienten hacia el germen causante: ma- El tratamiento debe estar guiado por los cultivos, tanto de o rin a como
yor incidencia de Pseudomonas en personas d iabticas y pacientes de de fluido obtenido por masaje prosttico, y prolongarse entre 4 y 16 se-
UVI, estafi lococo en adictos a drogas parentera les, Proteus en pacientes manas. El sndrome doloroso pe lviano crnico (SDPC) hace referencia a
con litiasis infectiva, presencia de sondas, catteres, tratamientos antibi- molestias genitourinarias de ms de tres meses de evolucin con cu ltivos
ticos previos, etctera. habituales negativos. Presenta etiopatogenia mu ltifactorial: infeccioso
por grmenes atpicos (urea plasma, micoplasma, chlamydias .. .}, psico-
En la evaluacin del paciente con pielonefritis y mala respuesta a trata- lgico (estrs), neurolgico (similar a fibromialgia y colon irritable). Trata-
miento inicial, es recomendable la realizac in de una ecografa para des- miento: antibiticos, antiinflamatorios, antidepresivos, rehabilitacin del
cartar obstruccin o litiasis. suelo plvico.

ITU en varones Se denomina prostatodinia a un cuadro clnico si milar donde predom i-


nan las molestias perinea les o gen itales con cu ltivos negativos y menos
Cualquier ITU en varn debe considerarse como complicada inicialmen- de 1Oleucocitos por campo en el lquido prosttico. Su causa es descono-
te, ya que hay que asumi r que existe afectacin del t ej ido prosttico, re- cida y el tratamiento difcil, emplendose actualmente a-bloqueadores o
nal o que existen problemas concom itantes como obstruccin urinaria, relajantes musculares como terapia in icia l.
litiasis o malformaciones urolgicas. Por todo el lo, el tratam iento debe ser
ms prolongado (mnimo una semana), no siendo adecuados los cursos Orquiepididimitis
cortos de tratamiento.
En varones menores de 35 aos se considera, en el plano terico, una en-
Prostatitis fermedad de transmisin sexua l, siendo los agentes ms frecuentes Chla-
mydia trachomatis y Neisseria gonorrhoeae. El tratamiento sera ceftriaxona
La infeccin aguda del t ejido prosttico se presenta como un cuadro sp- (1 25-250 mg) en dosis nica intramuscular ms 1Odas de doxiciclina oral.
tico con afectacin genera l del paciente, fiebre elevada, sndrome mic-
cional, artromialg ias y dificultad miccional (Tabla 3). En el examen rectal, En mayores de 35 aos el patgeno ms frecuente es el E.Coli y el trata-
la prstata aparece muy dolorosa e inflamada. El germen ms habitual es m iento sera con quinolonas o cefalosporinas 10-1 4 das
E. coli. Durante la inflamacin aguda, los antibiticos penetran adecuada-
mente, pero una vez que esta cede, la penetracin es ms pobre. Por ello, Absceso renal
se deben uti lizar cursos largos de tratamiento (3-4 semanas) para intentar
evitar la persistencia de focos que den pie a una prostatitis crnica. Entre Los abscesos medulares o cortica les suelen proceder de un foco de
los antimicrobianos empleados, las fluoroquino lonas son las que mejor pie lonefritis contiguo o de disem inacin hematgena de 5. aureus,
difu nden al tejido prosttico. procedente de focos cutneos en sujetos adictos a drogas por va

Lquido Cultivo lquido


Clnica Tratamiento
prosttico prosttico

Prostatitis E. coli Cuadro sptico + + Nunca hacer masaje prosttico ni sondaje Cotrimoxazol,
aguda fluoroquinolonas
4 semanas
Prostatitis E. coli Irritativo con + +1- > 1O leucocitos/campo + Cotrimoxazol,
crnica reagudizaciones, sin flu oroquinolonas
bacteriana 6-12 semanas
fiebre ni leucocitosis
Sndrome Desconocida/ Dolor genitou rinario Tipo A: inflamatoria: Antibiticos
doloroso Ureaplasma > 3 meses > 10 leucocitos/campo (grmenes
pelviano
crnico Mycop/asma Tipo B: inflamatoria: atpicos)
< 1O leucocitos/campo Rehabilitacin suelo
plvico
Antidepresivos

Tabla 3. Diagnstico d iferencial de las prostatitis Clasificacion NIH)


Manual CTO de Medicina y Ciruga, 2. a edicin

Etiologa Lesin tpica Diagnstico Tratamiento


Uretritis Neisseria gonorrhoeae Asintomtica ( f) Contacto < S das Cehriaxona
gonoccica Exudacin uretral matutina ( t) Gram de exudado cervical o espectinomicina
Epididimoprostatitis, sa lpingitis, sndrome Cu ltivo en medio de Thayer- (no en farngeas)
Fitz-Hugh-Curtis, gonococemia Martin Ciprofloxacino
diseminada (dficit C5-C 8, menstruacin,
embarazo, auxotipo AHU)
Uretritis no Ch/amydia trachomatis, Similar a las UG, pero con menos Contacto 7-15 das. Excluir gonorrea Tetraciclinas o macrlidos
gonoccica Ureap/asma urealyticum signos y sntomas por Gram y cultivo. C. inclusin-
Epididimitis, proctitis, cervicitis, EIP Giemsa IFD, medios celu lares

Tabla 4. Diagnstico diferencial de las uretritis

~~
parenteral. El urocultivo en este ltimo caso puede ser negativo. El
diagnstico m s fiable se realiza med iante TC. Deben tratarse con an- S. aureus es tambin la causa ms frecuente de endo-
tibiticos por va intravenosa y, depend iendo del tamao y de la evo- card itis infecciosa. ~
lucin, se hace obligatorio el drenaje med iante puncin percutnea o Recuerda
quirrgicamente.
El diagnstico es sim ilar al absceso renal, y su tratamiento pasa por el
Absceso perirrenal drenaje percutneo o quirrgico, con la adecuada cobe rtu ra antibitica.

Se loca liza entre la cpsula renal y la fascia de Gerota. Lo ms frecuente es ITU asociada a catteres
que un absceso cortical se abra a este espacio, pero puede ocurrir tam-
bin por diseminacin hematgena. El germen ms frecuente es E. coli, y La /TU es la infecc in hospitalaria ms frecuente, y los catteres urina-
S. aureus en los casos de diseminacin hematgena (Figura 1 ). rios la principal fuente de sepsis. Se ca lcu la que el 1% de catete ri smos
ambu latorios trans itorios sufren una /TU posterior y que la mayora de
pacientes con catter permanente presentan una bacteriuria significativa
al cua rto da de su colocacin. Esta bacteriuria puede hacerse sintomtica
en forma de cuadros de cistit is, hematuria o episodios feb riles, muchas
veces auto /i m itados.

Entre los factores que aumentan el riesgo de ITU asociada a catter urina-
rio se pueden enumerar: 1) sexo femenino, 2) edad avanzada, 3) mala tc-
nica de sondaje, 4) sistemas de drenaje abiertos y 5) falta de higiene local.

Entre los antibiticos disponibles, parece que las quinolo nas son los que
mejor elim inan la pelcu la biolg ica de los catteres infectados, favore-
ciendo as el tratam iento de la infeccin; en cualquier caso, este nica-
mente se recomienda si existe sintomatologa o en el momento de la
retirada del catter, por el mayor riesgo de ITU sintomtica y sepsis.

2.4. Tuberculosis genitourinaria

Genera lmente est ocasionada por Mycobacterium tuberculosis. El apara-


to genitou rin ario es el sitio ms frecuente de afectacin extrapulmonar
(tras la aden it is tubercu losa). Un 5% de los pacientes con tuberculosis
activa presentan afectacin del tracto genitourinario (Fi gura 2).

Tras la inhalacin del bacilo, se produce una diseminacin hematgena


(primoinfeccin) con siembra de bacilos en ambos riones en el 90% de
los casos. Sin embargo, la enfermedad clnica generalmente es unilateral.

El periodo de latencia entre la "siembra" y la enfermedad cln ica oscila en-


tre 1O y 40 aos, afectando principalmente a pacientes por debaj o de los
SO aos. La lesin inicia l m icroscpica se localiza en los g lomru los en for-
ma de granu lomas microscpicos. Al avanzar la enfermedad, se prod uce
afectacin ms dista l hasta la aparicin de una papilitis necrotizante, mo-
mento en el que ya puede existir paso de bacilos a la va excretora donde,
Fig ura 1. TC en la que se muestra un absceso perirrenal por procesos inflamatorios, ocasionar estenosis a nivel de los infund-

0 2 Infecciones del tracto urinario .


Cistitis intersticial
Urologa 1 02
bulos ca licia les, pelvis y urter, con hidronefrosis secundaria. La s lesiones ms evolucionado de la enfermedad, el rin puede encontrarse anula-
renales pueden cavita rse y calcificarse, y llegar a producir una destruccin do, dism inuido de tamao y con ca lcificaciones parenqu imatosas.
total del parnqu ima (fenmeno que se denom ina "rin mastic").
Tratamiento

El tratamiento mdico de la enfermedad activa no difiere sustancialmen-

Obliteracin
te del de la tuberculosis pulmonar en cuanto a frmacos y periodo de
ureteral tratamiento. Puede ser necesario el tratamiento quirrgico, dependiendo
de la complicacin asociada, genera lmente estenosis de la va excretora
e hidronefrosis. En caso de rin no funcionante por lesin extensa del
parnquima, puede ser precisa la nefrectoma.

Amputacin
calicial Rifampicina, isoniacida y pirazinamida son los tres
antibiticos ms emp leados en la tuberculosis.

Trompa

2.5. Cistitis intersticial

Aunq ue no es un cuadro infeccioso, se incluye en el presente captulo


esta entidad inflamatoria vesica l de origen desconocido. En este sentido,
se esgrimen dos teoras no demostradas: por un lado, la teora autoin-
Estenosis
ureteral distal munitaria, y por otro, la de un dficit en el recubrimiento urotelial por
glucosaminoglucanos.

Figura 2. Lesiones de la tuberculosis genitourinaria Clnica


Clnica Suele presentarse en mujeres entre 30 y 70 aos, como un cuadro cist-
tico crnico en el que destacan disuria, polaquiuria con nicturia y dolor
Los hallazgos cln icos son escasos. En el 70% de los pacientes, los snto- suprapbico, acompaados en ocasiones de hematuria (20-30%).
mas son leves. Lo ms frecuente es la aparicin de microhematuria, dolor
va go en flanco o clico renal. La afectacin vesical, sin embargo, s produ-
ce sintomatologa florida con un sndrome cisttico rebelde, donde lapo-
laquiuria (secundaria a la disminucin de la capacidad vesical) es lo ms Existen muchas ms causas de sndrome cisttico: cis-
t itis aguda, tuberculosis, carcinoma in situ, etctera.
llamativo. En varo nes, es frecuente la aparicin de una orquiepidid imitis
crnica que no responde a la terapia habitual.
Recuerda
En el90% de los pacientes, el an lisis urinario es anormal. Tpicamente apa-
rece piuria cida con urocultivo negativo. La prueba de laboratorio ms Diagnstico
importante es el cultivo de M. tuberculosis en medio selectivo (Lwenstein),
ya que los medios de tincin rpida (Ziehl, aura mina), aunque vlidos, pue- El diagnstico es bsicamente por exclusin de otra patologa que pueda
den dar falsos positivos por contaminacin con M. smegmatis. ocasionar un cuadro simi lar (infeccin bacteriana, tuberculosis, litiasis o
tumor vesical) apoyado en los hallazgos cistoscpicos sugestivos: 1) pe-
Diagnstico tequias submucosas, principalmente trigonales, que aparecen al d isten-
der la vejiga (glomerulaciones), 2) lceras de Hunner. La biopsia vesical,
El cultivo en medio de Lwenstein es positivo en el 90% de los pacientes adems de descartar la presencia de carcinoma in situ u otra patologa,
con enfermedad activa, aunque deben obtenerse, al menos, tres mues- revela en algunos casos, un infi ltrado intersticial de mastocitos.
tras de das diferentes para mejorar la sensibilidad, ya que el paso deba-
cilos a orina no es constante. Actualmente, lo ms rentable es realizar una Tratamiento
PCR de orina en busca del ARN del bacilo.
Aunque esta enfermedad raramente supone una amenaza para la vida
Radiolgicamente, el 90% de los pacientes presentan urogramas altera- de la paciente, su morbilidad es elevada. Desgraciadamente, las diversas
dos. El hallazgo ms sugestivo es la presencia de cavidades que comu- alternativas de tratam iento n icamente pueden encaminarse a una me-
nican con el sistema colector. Inicialmente estas cavidades son mnimas jora sintomtica, en la mayora de los casos con resu ltados discretos; 1)
y dan un aspecto "mordisqueado" a los clices. Segn la enfermedad distensin hidrulica vesica l, 2) amitriptilina ora l, 3) instilacin con dime-
avanza, pueden encontrarse estenosis infundibulares, ureteropilicas, en tilsulfxido [DMSO], 4) corticoides tpicos o sistm icos, 5) denervacin
un in ureterovesical o vejigas pequeas de aspecto rg ido. En el punto vesical, 6) cistoplastia s de aumento, y en ltimo trmino, 7) cistectoma.
Manual CTO de Medicina y Ciruga, 2 .a edicin

" El diagnstico definitivo de ITU es microbiolgico: ms de 1OS


Ideas clave J!6 UFC/ml. No obstante, este criterio vara con el sistema de reco-
gida.
" La causa ms frecuente de infeccin del tracto urinario (ITU) es
Escherich ia co/i, tanto a nivel comunitario como nosocomial. " Si se recoge la muestra urinaria mediante puncin suprapbica,
cualqu ier nmero de bacterias es significativo.
" El origen ms frecuente de uretritis es Chlamydia trachomatis.
" La bacteriuria asintomtica se trata en gestantes, menores de S
" La causa habitual de orquiepididimitis depende de la edad: aos, inmunodeprimidos, previamente a la ciruga urolgica, o
Chlamydia y gonococo si es menor de 3S aos; enterobacterias, si la especie implicada es Proteus.
si es mayor de esa edad.
" Los sistemas de drenaje cerrados son preferibles a los abiertos,
" La cau sa ms frecuente de absceso renal en el UDVP es Sta- pues la tasa de infeccin es menor.
phylococcus aureus.

3) Carcinoma renal.
Casos clnicos ~,/ 4) TBC urogenital.

Un prosttico, sin otros problemas de salud, portador de sonda RC: 4


uretral permanente, presenta bacteriuria (> 105 unidades for-
madoras de colonias) en dos urocultivos. Cul es la actitud tera- Una pacient e de 27 aos acude al servicio de Urgencias por do-
putica ms conveniente? lor en fosa renal derecha, fiebre de 39 oc,
escalofros y sndrome
miccional acompaante. Es alrgica a penicilinas. Seale la res-
1) Tratamiento antibitico de amplio espectro. puesta correcta:
2) Tratamiento antibitico segn antibiograma.
3) Cambio de sonda urinaria exclusivamente. 1) No ser necesario desca rtar patologa urinaria obstructiva en
4) Ant ispticos en vej iga urina ri a. este caso, ya que presenta un cl aro sndrom e miccional.
2) Para poder hacer el d iagnsti co de pielonefritis se deber cono-
RC: 3 cer primero los datos referidos a la fu nci n renal.
3) Se deber iniciar tratam iento em prico con un P-lactm ico.
Ante un paciente de 24 aos, que presenta fiebre alta con dolor, 4) Si en las primeras horas evoluciona favorablemente, podr con-
inflamacin y enrojecimiento testicular izquierdo, cul de las si- tinuar el tratamiento de forma ambulante.
guientes afirmaciones es INCORRECTA?
RC: 4
1) El diagnstico ms probable es el de epididimitis.
2) Los patgenos ms frecuentes son Ch lamydia trachomatis y Un pacient e de 83 aos sondado de forma permanente acude a
Neisseria gonorrhoeae. la consulta tras detectrsele dos cultivos positivos tomados con
3) El tratamiento de eleccin es va ncomicina + gentamicina. una semana de diferencia. Asegura encontrarse asintomtico. La
4) El trata miento de eleccin puede ser ofloxaci no. actitud ms adecuada ser :

RC: 3 1) Iniciar tratamiento antibit ico segn antibiograma de los culti-


vos obtenidos.
Ante un paciente que presenta febrcula persist ent e, crrsts 2) Tranquilizar al paciente y seguir con su pauta habitual de re-
renoureterales breves, piu r ia estril, orina con pH cido, mi- cambio de sonda.
crohematuria persistente, con citologa urinaria negat iva y 3) Realizar cambio de sonda de forma inmediata con tratam iento
epiddimos indurados en qu enfermedad se debe pensar antibitico.
primero? 4) Rea lizar ca m bio de sonda de fo rm a inmed iata con profil axis an-
t ibitica de 4 das.
1) Sarcoidosis.
2) Carcinoma vesica l. RC: 2

3) Long-term antibiotic t reatment is indicated.


Case Study 1 4) lt is un li kely that he needs to undergo a surgical procedure in
arder to solve his clinical synd rome.
A S2-year-old man presents to the emergency department with
complaints of fever, chills, perineal pain and pallor. In the mor- Correct answer: 2
ning, he had undergone a transrectal prostate biopsy. On physi-
cal examination, the patient presents persistent low blood pres- A 23-year-old woman sees the physician beca use a urine culture
sure and high fever. His laboratory tests reveal elevated WBC performed a week ago shows > 10,000 CFU/ml of non-resistant
count. Which of the following sentences is not correct? E. coli. Currently, she has no symptoms, and only claims that she
had cystitis once when she was a teenager. Blood tests reveal:
1) Hemodynamic stabilization of this patient is mandatory. creatinine 0.7, WBC count 9,000/mm 3 and hemoglobin 13.2.
2) Thi s clinica l condition is very ra re after a tra nsrecta l biopsy, sin- Pregnancy test is positive. Which of the following is the correct
ce it is nota frequent complication of this procedure. management in that case?

02 Infecciones del tracto urinario .


Cistitis intersticial
Urologa 1 02
1) Treatment is not necessary, sin ce it is an asymptomatic bacteriuria. Which of the following is the best diagnostic test for genitouri-
2) She will probably develop symptoms in a few days. nary tuberculosis?
3) She needs treatment, given the high likelihood of developing
acute pyelonephritis. 1) Agar-agar culture.
4) Ciprofloxacin 500 mg/ 12 hours may be a good treatment option. 2) Ziehi-Nielsen staining.
3) Urine PCR test.
Correct answer: 4 4) Lowenstein culture.

A 23-year-old woman, who is in her 13th week of pregnancy, is Correct answer: 3


brought to the emergency department complaining of hema-
turia, high fever, chills and left flan k pain. In the previous days, Which of the following clauses is not true regarding recurrent
she had presented dysuria, polyakyuria which reminded her of urinary tract infections?
cystitis. She had taken fosfomycin. In this patient, which of the
following options is correct? 1) lt is diagnosed w hen a patient presents four or more episodes
per year.
1) An abdominal x-ray must be performed in order to rule out an 2) lt usually appears in young male patients who are sexually ac-
obstructive cause. tive.
2) She may receive a 21 -day treatment with levofloxacin. 3) Sometimes it may be associated with Staphy/ococcus sapro-
3) A fetal wellness test must be performed . phyticus.
4) In pregnant women, cystitis can lead to acute pyelonephritis in 4) The first step in treatment management consists of hygiene
10% of cases. measures.

Correct answer: 3 Correct answer: 2


Urologi_a_

Uro litiasis

Tema fundamental en esta asignatura. Se debe conocer muy bien la actitud ante la litiasis
en general, ante los distintos tipos de clculos y, especialmente, todo lo relacionadocon el
tratamiento. Es un tema rentable y agradecido, as que hay que emplear el tiempo necesario.
La tabla-resumen de urolitiasis puede ser de gran ayuda.

3.1. Epidemiologa La enfermedad lit isica rec id iva en el 40% de los casos, con una media
de un nuevo clculo cada dos o tre s aos. Por rec idiva se entiende la
aparicin de una nueva litias is de la mi sma com pos icin y en la misma
La litiasis del aparato urinario es una patologa frecuente en la po blacin loca lizaci n, en un intervalo m enor de cuatro aos entre un c lcu lo y
mundial, y en Mxico representa un problema endm ico, const ituyendo otro.
una causa de consu lta y de atencin urgente con potencial dao estruc-
tu ral y repercusin funcional en todo el apa rato urinario.
3.2. Manifestaciones clnicas
Son numerosas las sust ancias q ue se han identificado formando parte de
los clcu los. Su incidencia vara segn el pas, e incluso segn las reas
y su manejo agudo
geogrficas dentro del m ismo pas.
El dolor agudo del clico renal es la manifestacin ms tpica de la litiasis
Se pueden d istinguir seis grupos de componentes: re nal. El dolor se produce por la sobred istensin de la va urinaria tras la
Oxalato c lcico. obstruccin de esta por el clculo. Es lgico, por t anto, que el clculo
Fosfato clcico. deba desplazarse desde su origen calicia l para producir sintomatologa
Fosfato no c lcico. aguda. Ocasionalmente se observan cuadros de dolor vago renal en rela -
Compuestos purnicos (cido rico, urato amnico, urato sdico, cin con litiasis ca liciales no desplazadas.
xantina, 2,8 dih id roxiadenina).
Am inocidos (cistina). El clico rena l o crisis renoureteral suele apa recer de forma progresiva
Otros (carbonato c lcico, su lfamidas, etctera) . sobre la fosa lumbar afectada, irradindose por el fl anco hacia la ingle y
los gen ita les (Figura 3). El paciente generalmente se encuentra afectado,
Los c lcu los de oxa lato clcico son los ms frecuentes, con cifras en torno con dolor que no cede con reposo, por lo q ue cambia de postura con-
al 65%, seguidos por los infectivos y cido rico (a lrededor del 15% cada t inuamente. Puede acompa arse de un cortej o veget ativo con nuseas,
uno). Fosfato clcico un 5%, y los de cistina con una incidencia baja (1-3%). vm itos y sudoraci n.

La terce ra dcada es la edad media de aparici n, por primera vez, de la El dolor irradiado hacia la ingle genera lmente indica que el c lcu lo ha
litiasis sa lvo en los de cistina, que suelen ser de aparici n ms prematura. alcanzado el urter. Cuando se encuentra en vecindad de la vejiga, o bien
dent ro de esta, puede aparecer un cuadro irritativo, similar al sndrome
La prevalencia estimada en la literatura es en torno al 10-15%. m icciona l con polaquiuria, d isuria y tenesmo vesica l.

Los c lcu los infectivos de estruvita, y en menor med ida, los de cido rico
y cistina pueden crecer modelando las cavidades rena les (litiasis cora li-
Las infecc iones urinarias son ms frec uentes en muje-
res que en varones. Por eso los c lcu los de estruv ita forme o "en asta de venado" [Figura 4]), manifest ndose no como clico,
tambin lo son. sino como infecciones urinarias de repet icin, dolor lumbar sordo, hema-
turia o incluso insufi ciencia rena l t erminal.
Urologa 1 03
La estruvita (o fosfato amnico magnsico) es el componente ms ca-
racterstico de los c lculos producidos por infeccin por grmenes urea-
lticos. Sus cristales t ienen form as prismticas polimorfas, y raramente se
observan los cristales "en atad" que pueden hallarse en el sedimento. La
cistina se reconoce fcilmente por su aspecto acaramelado, formando
crista les hexagonales en prismas o lminas.

En teora, el 90o/o de los clculos son visibles en una radiografa simple


RX simple de abdomen de abdomen, aunque este porcentaje es considerablemente menor en
con clculo a nivel de L3
las radiografas urgentes sin preparacin intestinal. Radiolgicamente,
la mayora de los c lculos son rad ioopacos, exceptuando los de cido
rico y algunas otras composiciones infrecuentes (su lfamidas, xantina,
indinavir).

El estudio de imagen se completar mediante otras tcn icas diagnsti-


cas. La ecografa permitir visualizar incluso las litiasis radiotransparentes,
con el inconveniente de no ser vistas aquel las ubicadas en el trayecto
Dolor irradiado Sndrome m iccional
Hemat uria ureteral (salvo las zonas cercanas a la vejiga o al rin). Tambin se podr
evaluar el grado de hidronefrosis.

La urografa ofrece informacin morfolgica y funciona l de ambos riones


(Figura 5). Debe tenerse en cuenta que, durante el clico renal, puede
Figura 3. Diagnstico de urolitiasis observarse una anu lacin funciona l, sin que signifique necesariamente
deterioro de dicha unidad renal. Mediante esta tcnica se puede diag-
nosticar todo tipo de clculos, ya sean rad iotransparentes o radioopacos.
El principa l inconveniente de este procedimiento es la introduccin de
contraste yodado, que est contraindicado en los pacientes con alergia,
creatin ina mayor de 2, mieloma m ltiple o deshidratacin importante.

Figura 4. Litiasis coraliforme o "en asta de venado"

Diagnstico

El an lisis bsico de ori na muestra genera lmente hematuria y leu-


cocituria. Una p iuria impo rtante apoyara la pos ib il idad de infeccin Figura 5. (a) Litiasis ureteral. (b) UIV de uropata obstructiva izquierda
sobreaadida, aunq ue ninguno de estos datos es rea lmente deter-
mi nante. Segn las guas clnicas, la urog rafa intravenosa (UIV) actualmente ha sido
desplazada por la TC helicoidal sin contraste, que se ha convertido en el
Los cristales de oxalato clcico dihidratado aparecen como bipirmides nuevo estudio de referencia para las litiasis. Aunque su alto coste hace que
tetragonales al observarlos con lupa binocular. Los de oxa lato clcico mo- todava no est extendido su uso, permite evaluar todo tipo de clculos.
nohidratado aparecen como cristales alargados que adoptan forma de
empalizada, formando clculos de estructura radiada, con aspecto com- En nios y embarazadas la ecografa urinaria es la tcnica de imagen de
pacto y macizo. Entre los fosfatos c lcicos, la brush ita es el compuesto eleccin.
ms cido, formando crista les grandes en fo rma de aban ico de color azu l
con luz polarizada. Las apati tas ti ene n aspecto microg ranu lar o esferocti- Tratamiento
co. El cido rico aparece bajo la lupa como una desordenada aglomera-
cin de cristales. En algunos c lculos, los cristales estn tan juntos que se El manejo agudo del cl ico renal se basa en el control del dolor. Para
asemejan a una masa continua. esto, es preciso conseguir una disminucin de la presin dentro de la
Manual CTO de Medicina y Ciruga, 2.a edicin

va urinaria, lo que puede hacerse, sobre todo, con antiinflamatorios, que 3.3. Evaluacin y tratamiento
dism inuyen el dolor y la diuresis al inhibir la sntesis de prostaglandinas.
Asimismo, se pueden usar espasmol ticos, que disminuyen la presin in-
de la litiasis renal
traureteral al relajar la pared del urter.
Este apartado se puede dividir en dos partes. Por un lado, el estudio de
Existen una serie de situaciones en las que el clico rena l se convierte en la litiasis con la finalidad de instaurar un tratam iento preventivo de su
una urgencia que precisa de hospitalizacin, y eventualmente, de mani- formacin, y por otro, el estudio y tratamiento de la litiasis ya formada.
pulacin invasiva:
Obstruccin grave, principalmente si se acompaa de litiasis mayor Estudio y tratamiento preventivo
de 10 mm.
Fiebre elevada (mayor de 38 C). La eva luacin del paciente con litiasis se basa en un estudio metablico
Dolor incontro lable. para determinar qu factores son modificables, en un intento de evitar la
Rin nico. recidiva (Tabla 5).

Asimismo, en pacientes diabticos, por el mayor ri esgo de co mp li-


cac iones, es aconsejab le, si no el ingreso, al menos una observacin 1

Litiasis clcica Litiasis


estricta. (oxalato o fosfato) rica

Una situacin similar ocurre durante el embarazo, donde una dilatacin Hipercalciuria Gota primaria Cistinuria Infecciones
leve de la va urinaria puede cons iderarse "fisiolgica", pero obstrucciones idioptica Hemopatas por grmenes
ms importantes o la aparicin de fiebre hacen aconsejable la colocacin Hipercalciuria productores
Enfermedades
secundaria de ureasa
de un catter ureteral (Figura 6 ). digestivas
a hipercalcemia
lngesta
Hiperuricosuria excesiva
Hiperoxaluria de purinas
Hipocitraturia Frmacos
Acidosis renal tubular Litiasis rica
distal idioptica
Litiasis c lcica
idioptica

Tabla S. Tipos de litiasis. Situac iones que favorecen su aparicin

Este estudio debe reservarse para aq uellos pacientes con alta probabili-
dad de recidiva, aunque cada vez ms autores ind ican que debe rea liza r-
se a todos los pacient es (Tabla 6).

La furosemida, al revs que las ti ac idas, aumenta el


ca lc io urinario .
----=--
Recuerda

Edad temprana de aparicin


Litiasis bi lateral
Litiasis en rin nico o malformado
Composicin poco frecuente
Litiasis recidivante
Nefrocalcinosis
Li tiasis co raliforme
Tabla 6. Pacientes con indicacin de estudio metablico

Desde el punto de vista prctico, las litiasis se pueden dividir en: las de
composic in c lcica y las de otras composiciones, ya que el primer grupo
supone la mayora de los casos (70-80%) tratados habitualmente.

Litiasis clcica

Figura 6. Doble J derecho. Litiasis uretera l derecha. Litias is coraliforme En la mayora de las ocasiones se desconoce el origen de la litiasis c lcica,
izquierda aunque se puede hacer una aproximacin a los facto res de riesgo que

03 Urolitiasis
Urologa 1 03
influyen en su aparicin. Slo en un pequeo porcentaje de casos existe Acidosis tubular renal distal (vase Seccin de Nefrologa). En-
una enfermedad de base q ue puede ser t ratada, y de esta forma desapa- fe rmedad autosm ica recesiva. Co nsiste en la imposibilidad del
rece la formac i n de c lculos c lcicos. tbulo d istal para excretar hidrogeniones a la orina (orinas persis-
Hipercalciuria idioptica. Es la ca usa ms frecuente de litiasis cl- tentemente alcalinas) con aumento de la el iminacin de calcio a
cica. Se defi ne como una excrecin urinaria de calcio mayor de 300 la orina.
mg/24 h en el varn y 250 mg/24 h en la mujer. De cara a su manejo, Existen formas incompletas que se observan en pacientes formado-
las tiazidas d ism inuyen el calcio urinario, reduciendo la formacin res de clculos de oxalato clcico y con hipercalciuria idioptica. En
de litiasis. La adm inistracin de citrato potsico ayuda a evitar la hi- estos probablemente la acidosis tubular no juegue un papel impor-
popotasemia y aumenta el citrato uri nario, que es inhibidor de la tante y responden a tiazidas.
litognesis (Tabla 7). Otras circunstancias que favorecen la litiasis clcica son: sarcoi-
dosis, sndrome de Cushing, d iuresis escasa, dficit de inhibidores o
anoma las en el pH urinario (a lcalosis).
Absortivas Resortivas Renales Litiasis clcica idioptica. Aproximadamente en el 20% de los pa-
Aporte excesivo Hiperparatiroidismo Acidosis tubular cientes con litiasis clcica no se demuestra ninguna anomala en el
Sd. de Burnett (leche Inmovilizacin distal estudio metablico.
y alcalinos) Tumorales ld ioptica
Hipervitaminosis D Enf. Paget
Litiasis rica
ldioptica Sd. de Cushing
Sarcoidosis El cido rico no disociado es poco soluble en orina. Con un pH urinario
de 5, la solu bilidad del cido rico es n icamente de 100 mg/1, m ientras
Tabla 7. Causas ms frecuentes de hipercalciuria que con un pH de 7 es de 1.580 mg/1. Esto demuestra la gran importan-
cia del pH urina rio en la formacin de clcu los de cido rico. Aparte
Hiperuricosuria. Excrecin en o rina de ms de 800 mg/24 h en el de estos, tambin existe una pequea proporcin de clculos de urato
varn o 750 mg/24 h en la mujer. Adems de favorecer la litiasis ri- monosdico y urato amnico.
ca, la hiperu ricosuria constituye un factor de riesgo pa ra la formacin
de clculos de ca lcio, probablemente por nucleacin heterognea El objetivo del tratam iento es reduc ir el cido rico excretado y aumen-
sobre ncleos de cido rico o urato sdico. Genera lmente se debe tar el pH urinario, ya que los clculos ms frecuentes en pacientes hipe-
a un exceso de pu rinas en la dieta. ruricmicos son los de cido ri co. Por otra parte, este t ipo de clculos
Hiperoxaluria. Se considera como ta l la excreci n en orina de son los que mejor responden al tratamiento mdico mediante quimi-
ms de 40 mg/24 h. Existe una hiperoxal uria pri ma ria, q ue es con- lis is por alcal inizacin urinaria. Pueden admin istrarse diversos lcalis; el
secuencia de un defecto enzimtico autosmico reces ivo; no t iene citrato potsico impedira el terico riesgo de formacin de clculos
tratamiento y genera lmente conduce a insuficiencia renal por litiasis clcicos por su efecto inhibidor, pero tambin pueden tratarse con bi-
recid ivante. El nico t ratamiento que existe actu almente es el tras- carbonato sdico o citrato sdico. Una alternativa es la acetazolamida
plante heptico, que suele ir unido al re nal, au nque algunos casos en dosis de 250 mg/da. Cua ndo, adems, la uricemia es alta, puede
responden a pi ridoxina . tratarse con alopurinol.

Litiasis cistnica
La ca usa ms frecuente de hiperca lcemia en un pa-
ciente ambul atori o es el hiperparatiro idismo prim a- La cistinuria es un trastorno autosm ico reces ivo en el que existe un de-
rio. En ca mbio, la hiperca lcem ia ms frecuente en fecto de absorcin, a nivel intestinal y tubular proximal, de los amino-
uno ingresado es la de o rigen neop lsico.
cidos dibsicos: cistina, ornitina, lisina y arginina (COLA), aunque parece
que puede existir un trastorno en el que nicamente se ve afectada la
No obstante, la mayora de los casos de hiperoxa lu ria son secundarios cistina, lo que indicara que, adems de un mecanismo de transporte co-
a malabsorcin de cidos grasos por enfermedades crnicas pancrea- mn, existe uno independiente para la cist ina .
tob il iares, derivacin intestinal para el tratam iento de la obesidad mr-
bida, resecci n ileal, enferm edad inflamatoria intestina l, hiperca lciuria Los niveles de cistina en orina de 24 horas son superiores a 100 mg, de
coincidente o por falta de ca lcio en la dieta, lo q ue perm ite que exista hecho, los homocigotos pueden excretar ms de 600 mg/da. El diagns-
mayor cantidad de oxa lato intestinal para su absorcin. t ico se realiza identificando los caractersticos cristales hexagonales en
La intoxicaci n por et ilengl icol y metoxiflurano puede producir hi- orina, o por una prueba positiva de nitroprusiato sdico (la orina se tie
peroxaluria, as como la ingesta de vitamina C en altas dosis. En todos de azul en pacientes afectados por esta enfermedad: test de Brand).
estos casos secundarios, el tratamiento con colestiramina, una dieta
pobre en grasas y la correccin de la malabsorcin, en la medida de El tratamiento consiste en aumentar la diuresis diaria (ms de 3 1/ da),
lo posible, suelen ser med idas eficaces. alcalinizar la orina por encima de 7,5 y, en caso de que esto sea insufi-
Hipocitraturia. Excrecin de citrato inferior a 300 mg/24 h. General- ciente, puede iniciarse tratamiento con D-pen icilamina (250 mg/6 h) o
mente se asocia a ot ra s ano malas urinarias. Aunque de causa des- a -mercaptopropion ilg licina (250 mg/6 h).
conocida, puede contribu ir una d ieta ri ca en protenas, hipoca liem ia,
enfermedad intest inal o infecci n uri nari a. Litiasis infectiva
Hiperparatiroidismo primario. Supone la causa ms frecuente de
hipercalciuria conocida (vase Seccin de Endocrinologa, metabolis- Los c lculos infectivos de estruvita o de fosfato amnico magnsico
mo y nutricin). (MgNH.P0 4 -6Hpl se desarrollan en un ambiente alcalino, producido por
Manual CTO de Medicina y Ciruga, 2 .a edicin

infeccin persistente de grmenes que hidrolizan la urea, aumentando A continuacin se anal izan brevemente las diversas fo rm as de trata-
la cantidad de amonio urinario. Los principales grm enes que poseen miento:
ureasa, adems de d iversas especies de Proteus, son Pseudomonas, Kle- Ciruga. Ha sido el tratam iento estndar hasta la aparicin de la
bsiel/a, Serratia y Enterobacter. La presencia de cue rpos extraos (sondas litotricia extracorprea. An hoy da, es preciso recurrir a la ciruga
vesicales, suturas) favorece su fo rm acin. cuando fracasan las ondas de choque o en determinados casos para
reducir la masa lit isica (c lculos cora liformes).
Para su tratamiento se han empleado diversos mtodos, generalmente Endourologa. La manipulacin endoscpica de la va urinaria es
ineficaces. La antibiotera pia nicamente mantiene estri l la orina durante cada da ms accesible gracias a las mejoras tcn icas. Puede realizar-
los cursos de tratam iento. se extraccin directa del c lcu lo mediante diversos tipos de pinzas o
cestillas, o bien fragmentar previamente el clcu lo med iante diversas
Parece ms prometedor el uso de inhibidores de la ureasa con cidos fuentes de energa, como la electroh idru lica, ultrasn ica o lser. Se
hidroxmicos. Estos son molculas anlogas a la urea que forman un puede acceder hasta el clculo mediante ureterorrenoscopia (URS) o
complejo enzima-inhibidor irreversib le. Se util izan bsicamente dos nefrolitotoma percutnea (NLPC).
sustancias de esta naturaleza : el cido propinico y el acetoh idrox- Litotricia extracorprea por ondas de choque (LEOC). Las on-
mico. das de choque se transmiten a travs de los tej idos corpo rales con
la misma impedancia acstica que el ag ua hasta alcanzar la litiasis,
Su empleo suele venir acompaado de cefa leas, temblores, trombosis sobre la que produce fenmenos de compresi n y descompres in
venosas u otros sntomas neurolg icos, por lo que tampoco son de g ran que conducirn a su fragmentacin .
aceptacin. Prcticamente todos los c lculos son susceptibles de tratamiento
mediante LEOC. La nica limitacin seran aquellos clculos no loca-
Todo lo relativo al estudio de la nefrolitiasis expuesto anteriormente se liza bies por su pequeo tamao(< 2-5 mm). Cualquier litiasis podra
puede repasar en la Tabla 8. ser tratada con LEO(, aunque esto tendr que ser matizado en fun-
cin de su tamao, composicin y dureza, loca lizacin, particularida-
Tratamiento de la litiasis des anatmicas de la va excretora y paciente, funcin rena l y tipo de
ya formada (Figura 7 litotriptor disponible.

La presencia de hipertensin arterial no controlada fac ilita el riesgo de


Los clculos ya formados no expu lsables (> 4-5 mm) precisan de trata- hemorragia durante la sesin de litotricia, luego deber ser estabilizada
miento "agresivo': es decir, necesitan ser extrados quirrgicamente o previamente a la m isma y constituye, en cierto modo, por ello, contra in-
fragmentados de forma que puedan ser expulsados espontneamente. d icacin relativa de LEOC (Tabla 9).

1
S . Estruvita (fosfato
a1es ca 1creas amnico magnsico) cido rico Cistina
1

Frecuencia Oxalato c lcico: 55-60% 10-15% 5- 10% 1-3%


Fosfato c lcico: 10-15%
Sexo Va rn Mujer Varn Va rn =Mujer
Etiologa Hipercalciuria idioptica Infeccin por grmenes ureasa Gota (50%) Cistinuria
ldioptica (+) ldioptica (<50%)
Hiperuricosuria (20%) Hiperuricem ias
secundarias
pH Alcalino Alca lino cido cido
Radiologa Radioopacos Radioopacos Radiotransparentes Radiolcidos
Morfologa Formas prismticas Aglomerados de crista les Cristales hexagonales
de los cristales polimrficas desorganizados, en prismas o lminas
Cristales "en atad" a veces formando masas
continuas
Cristales de fostato Ca

Cristales de OxCa Cristales de estruvita Cri stales de cido rico Cri stales de cistina
Tratamiento Hiperca lciuria idioptica: cido propinico y cido Alcalinizar la orin a Forzar diuresis (ingesta
tiazidas acetohidroxmico Alopurino l hd rica)
Hiperoxaluria 1.ia: piridoxina Antibioterapia (si hay hiperuricemia) Alca linizar orina
Hiperoxaluria 2.ia: En ocasiones ciruga Dieta de bajo contenido D-penicilamina
colestiramina proteico (si no hay respuesta)

Tabla 8. Tabla-resumen de las nefrolitiasis

03 Urolitiasis
Urologa 1 03
Manejo de la urolitiasis

Crisis Estable
cuadro agudo cuadro crnico

Edad?
Indican
Tipo de clcu los?
o no LEOC
Periodicidad de la clnica?
o ciruga
No complicado Complicado Tipo de sntomas?
Viabilidad renal?

Tratamiento sintomtico: Obstruccin grave


Espasmolfticos Tratar la condicin
Infeccin, fiebre Litotricia Cirug a
y antiinflamatorios preexistente
Dolor incoercible
Reposo e hidratacin Rin nico t
t
Ecografa renal
Clcica: acidificar (no til si oxalato).
citratos, tiazidas colestiramina
Extracorprea
(LEOC)
Ureterotoma
Pielolitotoma
y dieta baja en grasas y rica Percutnea Nefreoma
t en calcio, si hiperoxa luria
rica: alcalinizar, alopurinol
ultrasonogrfica
con microlumbotoma
Dilatacin
Estruvita: acetohidroxmico Endoscpica
Ingreso y tratamiento agresivo: Cistina: D-penicilamina, vit. B, va ureteral
Drenaje y alcalinizar
(catter o nefrostoma)
Contraindicada en:
Tratamiento parenteral:
Embarazo
- Antibiticos
Infeccin
-Remontar hemodinmica
Obstruccin distal
- Equilibrio electroltico
Aneurismas
- Narcticos
Coagulopatas
Obesidad
Vigilancia estrecha
Arritmia cardaca

Figura 7. Manejo de la litiasis renal ya formada

Relativas (precisan de control Esta posibilidad es mayor ante litiasis de gra n tamao, por lo que en al-
Absolutas
1
previo al tratamiento) gunos de estos casos se puede colocar un catter de derivacin urinaria
Embarazo Alteraciones de la coagulacin (nefrostoma o doble J) antes de la LEOC para disminuir este riesgo, ge-
Obstruccin distal Aneu ri sma artico neralmente en litiasis superiores a 2 cm.
Infeccin activa Alte raciones del ritmo cardaco,
marca pasos o desfibriladores Derivadas del efecto directo de las ondas de choque, pueden apa recer
Obesidad contusiones renales manifestada s como hematuria, hematomas rena les,
Hipertensin arteri al descontrolada equimosis o eritem a cutneo, y en grado mximo, rotura re nal. La hema-
Tabla 9. Contraindicaciones de la LEOC turia se considera la complicacin ms frecuente de la litotricia.

Complicaciones Ms controvertida es la terica relacin de la LEOC con la aparicin de


hipertensin arterial, ya que no est demostrada en las ltimas revisiones
La expulsin de fragmentos litisicos puede ocasionar un clico renal y, publicadas, aunque s la relacin entre hematoma renal post-LEOC e hi-
con menor frecuencia, obstruccin ureteral (steinstrasse o "calle lit isica"). pertensin arterial.

" Los clculos asociados a las resecciones ileales o a la enferme-


Ideas clave Ji5 dad inflamatoria intestinal son de oxalato clcico.

" Los clculos ms frecuentes son los de oxalato clcico. " Precipitan en medio cido: cido rico y cistina. Precipitan en
medio alcalino las que contienen fosfatos (fosfato amnico
" Globalmente, la litiasis es ms comn en el varn, salvo las de magnsico o estruvita, y el fosfato clcico) .
estruvita, ms comunes en mujeres.
" En el tratamiento de la litiasis por cido rico es beneficioso al-
" La radiografa de abdomen no permite ver algunos clculos, calinizar la ori na.
como los de urato. Sin embargo, la ecografa puede verlos, inde-
pendientemente de su composicin. " Los clculos de oxalato NO se ven alterados por el pH (al Oxa l,
el pH de la igual).
" Litiasis rad iotransparentes: Sulfamidas, lndinavir, Urato, Xanti-
nas (SIUX). Las de cistina son radiolcidas; y el resto, radioopa- " Los clculos de estruvita se relacionan con microorganismos
cas. productores de ureasa, como Proteus.

" Las tiazidas son tiles para la hipercalciuria idioptica. " La s contraindicaciones absolutas para la LEOC son: embarazo, in-
feccin activa y obstruccin de las vas urinarias distal al clculo.
Manual CTO de Medicina y Ciruga, 2.a edicin

cias por dolor en fosa renal izquierda de cinco das de evolucin,


Casos clnicos asociado en las ltimas 24 horas a fiebre, escalofros y malestar
general. Analtica de sangre: plaquetopenia, leucocitosis y dis-
A un hombre de 29 aos, con antecedentes de dolor tipo clico minucin de la actividad de la protrombina. Analtica de orina
en fosa renal izquierda que cedi con tratamiento analgsico, se normal. Radiografa de abdomen con claras imgenes de litiasis.
le practica una urografa intravenosa, aprecindose defecto de Eco renal: dilatacin moderada de sistema excretor izquierdo.
replecin radiotransparente de 6x7 mm en tercio distal de ur- Cul es la conducta ms adecuada?
ter izquierdo. El pH de la orina fue de 5,5; asimismo, se observan
cristales de urato, 9-12 hemates por campo y escasa leucocitu- 1) Solicitar hemocultivos y urocultivo para establecer la necesidad
ria. Cul sera el tratamiento ms apropiado? de antibioterapia .
2) Realizar urografa intravenosa para intentar filiar la causa .
1) Alopurinol va oral. 3) Hidratar a la paciente bajo observacin rigurosa, y repetir eco-
2) Ureteroscopia con extraccin del clculo. grafa a las 48 horas.
3) Nefrolitotoma endoscpica percutnea. 4) Colocar catter doble J o practicar nefrostoma percutnea de
4) Alcalinizacin de la orina por va oral. forma inmediata con cobertura antibitica.

RC: 4 RC: 4

Mujer de 50 aos, diabtica insulinodependiente, con infeccio-


nes urinarias y clicos nefrticos de repeticin. Acude a Urgen-

A 34-year-old woman is hospitalized with complaints of right


Case Study flan k and iliac fossa pain that commenced seven hours ago. On-
set was sudden, and the pain is slightly irradiated to her genitalia.
A 28-week pregnant patient is brought to the emergency de- She also presents sweating, nausea, vomits and chills. Her phy-
partment with a sudden onset of severe left flank pain irradia- sical examination reveals body temperature 36.8C and blood
ted to her groin. No dysuria, nor in the previous days. She also pressure 120/75. A plain abdomen x-ray film shows a calculus.
presents nausea and vomiting. No fever. No similar previous epi- After analgesic treatment, the problem seems to be resolved and
sedes are recorded. Which of the following clauses is the most she is discharged; 1O days later, she comes to your outpatient
appropriate? clinic with the results of an intravenous urography that reveal
delayed elimination of contrast throughout the right kidney
1) The first test to perform is a plain x-ray film, sin ce it will enable with a ureteric grade 11 proximal dilation. A 6 mm calculus is also
us to see a lithiasis, if present. observed at the end of the dilated segment. Laboratory studies
2) lt would be reasonable to think that the patient's symptoms are do not show any abnormal parameters. Which of the following is
consistent with a lithiasic reno-ureteral colic, since it is the most the recommended attitude in this case?
common urinary complication in pregnant patients.
3) The first diagnostic test to perform is a contrast-enhanced CT 1) An emergent urinary referral must be performed.
sean, the new gold standard for diagnosis of urinary lithiasis. 2) Perform a scheduled ureterol ithotomy.
4) Ultrasound study may be used in order to analyze whether or 3) Conservative management may be tried first, by administering
nota urinary derivation device would be indicated. propionic acid.
4) The best treatment in this case may be extracorporeal shock
Correct answer: 4 wave lithotripsy (EWSL).

A 34-year-old woman is admitted to our emergency department Correct answer: 4


with a history of right flan k and right illiac fossa pain for the last se-
ven hours. The pain onset was sudden and she has also presented A patient presents to the emergency department with a history
sweating, nausea, vomiting and chills. Physical examination revea- of sudden pain after receiving extracorporeal shock wave litho-
led temperature 36.8C and blood pressure of 120/75. Which ofthe tripsy to treat a ureteric calculi. What is the most frequent com-
following da uses about this patient is correct? plication of this procedure?

1) The first diagnostic test to perform in this patient is a contra st- 1) Hematuria.
enhanced CT sean. 2) Steinstrasse.
2) ln itial treatment should in elude urine alkalinizing agents. 3) Hypertension.
3) Plain x-ray film will show typical staghorn calculus. 4) Renal hematoma.
4) The first step in treatment is administration of analgesic agents.
Correct answer: 1
Correct answer: 4

03 Urolitiasis
_Uralngja _____ _____

Tumores renales

ORIENTACIN Asimismo, existe una incidencia aumentada en el rin poliqustico, en


Es fundamental reconocer el
ENARM sndrome de Stauffer. la enfermedad qustica adquirida de la insuficiencia rena l crnica y en los
riones malformados, como el rin "en herradura':

Procede de las clulas del tbulo contorneado proximal, y microsc-


picamente predominan las clulas claras sobre las granulares y fusi-
4.1. Carcinoma de clulas renales fo rmes.

( adenocarcinoma renal, hipernefroma)


La esclerosis tuberosa y la enfermedad Von Hippei-
Es el tumor slido renal ms frecuente (90%) (Figura 8). Es un tumor fun- Lindau se asocian tambin a otro tumor renal, el an- lr""";;::a,;"!"\.;o<:!
damentalmente de la edad adu lta, con mayor incidencia entre los 40 y 60 giomiolipoma.
aos, con predominio en el varn 2:1 a excepcin de la variedad crom-
faba, tpica de las mujeres.
Presentacin

La trada clsica: hematuria, dolor y masa en el flanco ocurre nicamente


en ell 0% de los casos y, cuando se presenta as, generalmente se trata de
una enfermedad avanzada.

Actua lmente la mayora de las neoplasias renales son descubiertas inci-


dentalmente en pruebas de imagen de rutina por otra causa.

La anomala ms frecuente es la hematuria macroscpica o microscpi-


ca (60%). Otros hallazgos frecuentes son dolor (40%), prdida de peso
(30%), anemia (40%), masa en flanco (24%), HTA (20%), hiperca lcemia
(6%), eritrocitosis (3%). El 20% de los pacientes presentan como cuadro
paraneoplsico alteracin de las enzimas hepticas sin evidencia de me-
tstasis (sndrome de Stauffer).

Ocasionalmente, el adenocarcinoma renal puede producir hormonas


productoras de sndromes clnicos segn la sustancia secretada. Entre
estas se encuentran pptidos PTH-/ike, prostaglandinas, prolactina, reni-
na, gonadotropinas o corticoides. La invasin de la vena renal principal
en el lado izquierdo puede ocasionar la aparicin de un varicocele de
Figura 8. Carcinoma de clulas renales forma repentina, que no disminuye en decbito.

Entre los factores de riesgo que se han implicado se encuentran el humo


del tabaco, el cadmio la obesidad y la HTA. Existen formas familiares que
La produccin de pptidos puede aparecer en el hi-
suelen ser mltiples y bi laterales, como en la enfermedad de Von Hippei- pernefroma, pero es ms tpica de carcinomas epider-
Lindau y, en menor medida, la esclerosis tuberosa. Se han identificado moides (pulmn, esfago, etctera) .
alteraciones cromosm icas que implican al cromosoma 3.
Manual CTO de Medicina y Ciruga, 2. a ed icin

Diagnstico

La ecografa es la primera prueba complementaria que debe real izarse,


de forma que la identificacin ecog rfi ca de una lesin que cumple cri-
terios de qu iste simple (contorno li so, conten ido transnico y refuerzo
posterior) hace innecesarios mayores esfuerzos d iagnsticos, pudiendo
hacerse un seguimiento ecogrfico anual. De esta forma se diagnostican
la mayora de las masas renales en la actualidad (Figura 9).

Figura 11. RM de tumor renal con trombo en venas renal y cava

Aunque la urografa intravenosa (UIV) contina siendo la base del d iag-


nstico por imagen en urologa, en el caso del adenocarcinoma renal
proporciona pocos datos y de fo rma indirecta, como puede ser la d istor-
sin del sistema co lector, su ocupacin o la anulacin func iona l del rin.
En las placas tomogrficas de la UIV puede observarse la presencia de
una masa o una alteracin del contorno renal.

La arteriografa renal, exploraci n obligada hace aos, ha quedado rele-


Figura 9. Ecografa de quistes renales simples gada a los casos dudosos, riones nicos y otras situaciones en las que
se plantea tratamiento quirrgico conservador. El patrn arteriogrfico
La realizacin de puncin-aspiracin con aguja fina (PAAF) de una masa caracterstico incluye neovascularizacin tumora l, lagos venosos, fstu las
renal para su filiacin es una exploracin agresiva que, debido a su baja arteriovenosas y vasos capsulares.
sensibilidad, no se justifica actua lmente, excepto en casos excepciona les.
El estudio de extensin, si se sospechan metst asis, se completar rea li-
La tomografa computarizada (TC) es el mejor mtodo aislado para eva- zando radiografa de trax, analtica heptica completa y, en algunos ca-
luar una masa renal, proporcionando informacin precisa sobre metsta- sos dudosos, gammagrafa sea.
sis ganglionares (80%) y afectacin de rganos adyacentes (Figura 10).
El proced im iento d iag nstico ante la presencia de masas rena les se pue-
de observa r en la Figura 12.

Masa renal descubierta accidentalmente

!
Examen fsico
Anlisis de orina

..
Quiste complejo
o masa slida

~
Observar

~--~
M~a- --------
sa_s_~li~
d-a --~ ~Angiomiol ipoma
Figura 1O. TC de masa renal izquierda o quist e complicado

La RM, aunque no se emplea de manera rutinaria en este t ipo de pacien-


Nefrectoma No complicado: Complicad o:
tes, s se uti liza como mtodo de diagnstico bsico en sujetos en los radical observacin Nefrectoma simple
que se sospecha afectacin trombtica tumora l de la vena re nal o cava o parcial Nefrectoma parcial
(Figura 11 ). Embolizacin

Figura 12. Algoritmo diagnstico de las masas renales

04 Tumores renales
Urologa 1 04
Tratamiento seg unda lnea. Las situaciones qu e favorecen la respuesta al t rat amiento
inmunoteraputico son: presencia de metstasis pulmonares exclusiva-
Una vez estudiado el tumor y descartada la prese ncia de metst asis, mente, buen estado general, y que se haya rea liza do la nefrectoma antes
ta nto viscerales como gang lionares, el tratam ient o de eleccin es la del descubri miento de las masas pulmonares.
nefrectoma radical, incluyen do la fa scia de Gerot a y la g lndu la su-
pra rrenal. En la actualidad se investiga sobre autovacunas elaboradas con linfocitos
peritumora les que parecen ofrecer resultados alentadores.
En ocasiones especiales se puede plantea r la cirug a conservadora o par-
cial, como en los tumores bilaterales, en aquellos que aparecen sobre ri-
n n ico, o suj etos con nefropatas md icas, en los q ue la prdida de 4.2. Otros tumores
masa nefro nal obliga ra a dilisis.

Adems de en est as indicaciones imperativas, actualmente es el nuevo Tumor de Wilms (vase Seccin de Pediatra).
patrn de referencia en el t ratam iento de tu mores pequeos (menos de Tumores renales metastsicos. Pueden encontra rse metstasis en
4 cm), bien deli mitados y sin afectacin de la grasa perirrenal. En estos el rin de tumores de pu lmn (la ms frecuente), mama, metano-
pacientes seleccionados pa rece que la supervivencia y la tasa de recidivas mas e infiltracin por linfoma.
locales son semej antes a las que se present an en casos sim ilares tratados Tumores benignos:
con nefrectoma radical. Adenomas corticales: so n los tumores ms frec uentes del
adulto, aunq ue indistin g uib les clnica mente del adenocarci-
La linfadenectoma regional no mejora la supervivencia y nica mente noma, por lo q ue se t ratan como tales. El criterio clsico de
tiene, por ta nto, va lidez en la estadificacin, por lo que no se rea liza siste- ta mao (3 cm) para su diagnsti co d iferencia l no es v lido en
mtica mente. Este se establece sobre la base de los hallazgos quirrgicos la actualidad.
y anatomopatolg icos. Angiomiolipomas: se asocian a la esclerosis tuberosa en un
SOo/o. Compuestos de una proporcin variable de grasa, vasos
Otras fo rmas de t rata miento ca recen de eficacia. Tanto la quim ioterapia y fibras muscu lares. Cuando son gra ndes (mayores de 4 cm},
como la rad ioterapia ofrecen resultados pobres. En el caso de enferme- pueden ocasiona r un snd rome de Wnderlich por sangrado
dad metastsica, las opciones son mltiples, pero ni ng una sati sfactoria. retroperitoneal. Cuando se asocian a esclerosis tuberosa, suelen
Aunque se ha descrito la regresin de las lesiones meta st sicas tras la ser m lt iples y bilateral es, por lo que deben trata rse de forma
nefrectom a, esto ocurre nica mente en un 1o/o, y generalmente de forma conservadora.
tra nsitoria, por lo que no se ju stifica, sa lvo de forma pa liativa por ot ros Oncocitoma: co nsid erado be nig no, aunq ue en algunos se
motivos. han detectado metst asis. Hay criteri os radio lg icos para dis-
t ingu irlo del ade noca rcin o ma, pero en la mayora de los casos,
La inmunoterapia con interfero nes, interleucina, li nfocit os killer activa- ni estos ni la citologa o la biopsia, ofrecen ga rantas suficien-
dos, y ciertas combinaciones de qu imioterapia con inmunoterapia, son tes de su benig nidad, por lo q ue tienden a ser tratados me-
altern ativas para la enfermedad metastsica, pero en ning una de ellas se diante nefrectoma.
obtienen tasas de respuesta su periores al l So/o. Actu almente se utiliza n Nefroma mesoblstico (hamartoma fetal): es el t umor be-
de preferencia: antitirosi ncin asas en primera lnea, antiangiognicos en nigno ms frecuente en reci n nacidos y lactantes.

" El hipernefroma puede producir multitud de sndromes para-


Ideas clave S neoplsicos. Esto puede compl icar bastante el diagnstico, de
ah el sobrenombre de "tumor del internista".
" El ms frecuente de los tumores slidos renales es el hiperne-
froma . " La elevacin de las transaminasas sin afectacin heptica es t -
pica del hipernefroma (sndrome de Stauffer).
" El paciente caracterstico es un varn de mediana edad, obeso
y fumador. " No se debe confundir un quiste simple con un hipernefroma.
Los criterios de quiste simple son: contorno liso, contenido tran-
" La trada clsica consiste en hematuria, dolor y masa en flan - snico y refuerzo posterior.
co, actualmente, lo ms habitual es que sea incidentaloma
(asintomtico) . Si produce sntomas, el ms frecuente es la " La primera prueba de imagen, ante la sospecha de hipernefro-
hematuria. ma, sera la ecografa.

" Hay que sospechar tumor renal ante un varicocele izquierdo, de " El t ratamiento fund amental del hipernefroma es la extirpacin
aparicin sbita y que no cede con el decbito. quirrgica. La q uimioterapia y radiote rapia tienen un papel muy
secundario.
Manual CTO de Medicina y Ciruga, 2. 8 edicin

Un paciente de 62 aos, con alteracin de la funcin renal y crisis


Casos clnicos . de hematuria, presenta una masa abdominal palpable en flanco
derecho. Se le realiza una TC, detectndose una masa de carcter
Un hombre de 45 aos tiene un carcinoma de clulas renales ex- slido de 8 cm de dimetro en rin derecho. En la anamnesis
tendido. Los niveles de GOT, fosfatasa alcalina, LDH y a-2 globu- destaca que el paciente es fumador de 35 cigarrillos al da. Cul
lina son elevados y el tiempo de protombina alargado. El hgado es, entre los siguientes, el diagnstico de presuncin ms pro-
aparece difusamente agrandado, pero no existen defectos foca- bable?
les de infiltracin intraheptica. La explicacin etiolgica ms
probable para estos hallazgos ser: 1) Nefroblastoma.
2) Liposarcoma .
1) Los efectos hepatotxicos de tumor. 3) Angiomiolipoma .
2) Metstasis heptica. 4) Adenocarcinoma.
3) Amiloidosis.
4) Trombosis tumorales que obstruyen la vena heptica. RC: 4

RC: 1

A woman presents to the clinic with a three-month history of


Case Study . flan k pain. She feels she has a mass in that area and has lost 12 kg
in two months. Her urine is darker than usual and she has seen
A 62-year-old patient comes to the hospital wth complaints small amounts of blood in her urine sometimes. Regarding this
of difficult micturition, weak voiding and postvoid drbblng. case, which of the following is correct?
Ultrasound study reveals a 3 cm-diameter hypoechoic mass
in the right kidney. Posterior acoustic enhancement s also 1) The m ain diagnostic suspicion is renal adenocarcinoma and the
observed. Left kidney appears to be normal. In ths case: prognosis is good since she presents with initial symptoms.
2) The main diagnostic suspicion is renal adenocarcinoma and in-
1) A fine needle aspiration biopsy shou ld not be performed. travenous urography should be performed in order to complete
2) An ultrasound-guided fine needle aspiration biopsy should be the diagnostic study.
performed. 3) You should suspect a xanthogranulomatous pyelonephritis. Ne-
3) A CT-scan guided fine needle aspiration biopsy should be per- phrectomy is indicated according to the patient's symptoms.
formed. 4) The main diagnostic suspicion is renal adenocarcinoma and an
4) A tru-cut biopsy should be performed. abdominal CT sean should be performed in order to study the
extension of the disease.
Correct answer: 1
Correct answer: 4
An ultrasound shows a right kidney mass that s 3 cm n diameter
and apparently sold. Left kidney seems to be normal. A CT sean A right renal mass measuring more than 7 cm is incidentally
is performed and it shows that the mass is located in the superior found during a routine ultrasound. Lab tests show: creatinine
half of the right kidney and it has a fat-like density, suggesting 0.95, Na: 138, K: 4.1, GOT 293, GCP 324, AF 842, BT 0.95, Hb 14.2,
an angiomyolipoma. Which of the following clauses is incorrect white blood cell count 9,600, platelets 180,000. In this case:
regarding this case?
1) CT sean will probably disclose metastatic liver lesions.
1) We can put the patient atea se regarding their renal mass. 2) These findings are dueto liver compress ion by the renal mass,
2) A karyotype should be performed in order to rule out tuberous since it is located within the right kidney.
sclerosis. 3) The renal function parameters may lead us to considera parta!
3) Annual follow-up with CT sean cou ld be performed. neph rectomy despite the tumor's size.
4) lt could cause Wunderlich syndrome. 4) The hepatotoxic effect of the tumor usually disappears after the
nephrectomy.
Correct answer: 2
Correct answer: 4

04 Tumores renales
_Urologa

Hiperplasia y carcinoma
, .
prostat1co

ORIENTACIN Tanto la hiperplasia p rosttica benigna como el cncer de prstata son dos temas fundamentales.
Probablemente el cncer sea ms importante, sobre todo en lo referente al tratamiento.
ENARM Hay que aprenderse muy bien el resumen de la Tabla 11.

5.1. Hiperplasia prosttica benigna etiopatogenia no est clara; aunque el estmulo andrognico a travs de
su forma activa, la dihidrotestosterona, es fundamental, su papel exac-
La hiperplasia prosttica benigna (HPB) afecta en mayor o menor grado to no ha sido determinado. Las teoras ms recientes abogan por un
a la gran mayora de los va rones a pa rti r de la qu inta dcada de la vida, desequi librio hormonal de estrgenos/andrgenos, o por la existencia
alcanzando el 80-95% de la poblacin mascul ina de 80 aos. de factores de crecimiento prostticos con un papel permisivo del am-
biente hormonal.
La prstata se divide clsicamente en cinco lbu los (anterior, medio, pos-
terior y dos laterales); aunque estos nicamente se encuentran como No existe evidencia de asociacin entre HPB y carcinoma prosttico.
tales en la edad fetal. En el adulto se puede interpretar la anatoma de
la prstata dividida en dos partes: una zona perifrica, donde se origina Diagnstico
principalmente el carcinoma, y una zona periuretral o transicional, de la
que procede la HPB (Figura 13). El crecimiento prosttico genera lmente se produce hacia la uretra, oca-
sionando obstru ccin de esta y dificu ltando el vaciam iento vesical (Figu-
La HPB est compuesta de una proliferacin variable de elementos ra 14). Esto no se manifiesta inmediatamente, sino que, genera lmente, el
glandulares, musculares y del estroma, que en su crecimiento compri- proceso pasa por una serie de etapas que incluyen una fase de compen-
men la prstata perifrica, formando la llamada cpsula quirrgica . Su sacin, una fase cln ica y una de descompensacin.

Lbulo anterior

Zona de tran sicin

latera l
~ Lbulo posterior

Glndu las
suburetra les

Figura 13. Anatoma de la prstata


Manual CTO de Medicina y Ciruga, 2 .a edicin

Fase de descompensacin (Tabla 10). Se produce un venci-


miento del detru sor vesica l, que es incapaz de vencer la presin
uretra l, aumenta nd o la si ntom atolog a anterio r y p ud iendo pre-
se ntarse rete nci n urinaria. Ocas iona lmente puede apa recer d il a-
tac i n ureteral bil at eral con deterioro de la fun cin renal. Esto se
debe a uropata obst ructiva infravesical con prdida del mecanis-
mo antirrefluj o.

Sntomas
Sntomas Sntomas de llenado
obstructivos
postmiccionales o "irritativos"
o de vaciado
Dificultad de inicio Sensacin Urgencia miccional
miccional de vaciado Nocturia
Disminucin incompleto
Polaquiuria diurna
del cal ibre y fuerza Goteo
Dolor suprapbico
del chorro miccional postmiccional
Figura 14. Ecografa de hiperplasia prosttica con crecimiento del Incontinencia
Miccin
por urgencia
lbulo medio intravesical intermitente
miccional
y prolongada
Fase de compensacin. El crecimiento prosttico ocasiona un au- Retencin de orina
mento de la presin uretral durante el vaciado que es compensado e incont inencia
por rebosamiento
por una mayor actividad contrctil del detrusor q ue se hipertrofia,
encontrand o presiones vesica les ms elevadas. En esta fase, la clnica Tabla 10. Sntomas HPB
puede ser mnima o inexistente.
Fase clnica (Figura 15). La elongacin de las fibra s musculares por Pueden producirse tambin otro t ipo de sntomas denom inados de lle-
encima de un lmite condiciona prd ida de capacidad contrctil. En nado o "i rritativos", que son debidos a la alteraci n funciona l vesical, y
este momento aparece retraso de l inicio de la miccin, dism inucin cuya resolucin es ms d ifcil tras la desaparicin de la obstruccin. En-
del calibre y de la fue rza del chorro miccional y alargamiento del va- tre estos sntomas se incluyen polaquiuria, tenesmo, nicturia y urgencia
ciado (lo que en conjunto se denomina sntomas obstructivos). El va- micciona l. La HPB es la causa ms frecuente de obstruccin del tracto
ciado suele ser incompleto, dando lugar a un res iduo posmicciona l. urinario inferior en el va rn.

En la eva luaci n de l sndrome prosttico, el tacto recta l contina sien-


do la ex plora ci n fund am ental, sob re t odo pa ra d iferenciarlo del car-
cinoma, ya que no es infrecuente q ue am bas entid ades coexista n. La
clni ca es lo m s importante para va lora r la indica cin de tratamiento
de la HPB, ya que no existe correlacin entre el tamao prosttico y
el grado de obstruccin. Cualquier zona sospechosa al tacto debe ser
biopsiada.

La med icin del fluj o m ximo micciona l es tambin importante, con-


sider ndose norma l cuando es mayor de 15 ml/s y claramente pato-
lg ico si es menor de 1O ml/s. El estudi o puede comp letarse con una
ecografa que permita eva luar si existe afect ac i n del tracto urinario
superi o r, residu o posm iccional, liti asis vesica l u otra patologa asociada .
El uso del PSA en la HPB nica mente est ind icado para desca rtar la
presencia de carcinoma en la prstata, ya que no sirve para d iagnosticar
HPB, au nque rec ientemente ha demostrado ser el mejor pred ictor de la
historia natural de la enferm edad . Es decir, que mayores niveles de PSA
en HPB d iagnosticada probablemente se correlacionarn con mayores
vo lmenes prostticos y con ms posibi lidades de comp licacin deri-
. vadas de la HPB.

Tratamiento
Dentro de las posibilidades terapu ti cas, la ciruga co ntina siendo el
nico tratamiento definitivo para la HPB. Esta puede ser endoscpica
(RTUp: resecc in transuretra l prosttica) o abierta (adenomectoma
Urografa intravenosa mostrando impronta prosttica en vejiga (vejiga "en prosttica) (Figura 16), dependiendo del tamao de l adenoma. En el
montera") con urteres en anzuelo 10% de las piezas obtenidas se encontrarn focos de adenocarcinoma
Figura 15. Hiperplasia prosttica en fase clnica incidental.

05 Hiperplasia y carcinoma prosttico


Urologa 05
5.2. Carcinoma prosttico

El adenocarcinoma prosttico es el tumor maligno ms frecuente del apa-


rato genitourinario mascu li no y el segundo en frecuencia general, despus
del pulmonar. Sin embargo, si se incluyesen los carc inomas incidentales y
los encontrados en autopsia, supera al pulmonar en preva lencia.

La hormonodependencia del cncer prosttico parece indicar el papel


de los andrgenos en su etiologa o patogenia. La relacin de factores
genticos, ambientales o infecciosos no ha quedado suficientemente es-
tablec ida.

El 95% de los carcinomas prostticos son adenocarcinomas originados


en la zona perifrica de la prstata. Los carcinomas ductales se originan
en los conductos prostticos en luga r de los acinos, e histolgicamente
pueden corresponder a carcinomas transicionales, escamosos, endome-
trioides o mixtos. Ms raros son los carcinosarcomas (menos del 1%).
Figura 16. Adenomectoma prosttica
El adenocarcinoma prosttico, con frecuencia, es mu ltifocal y presenta po-
Se debe tener en cuenta que en la ciruga de la HPB no se extirpa la cp- blaciones en distinto grado de diferenciacin. En esta heterogeneidad se
sula quirrg ica, que est co nstituida por las glndulas prostticas perif- basa la clasificacin de Gleason, que asigna una puntuacin de 1 a 5, segn
ricas comprimidas por el adenoma, y es el principal origen del carcinoma el patrn histolgico de cada una de las dos poblaciones ms representa-
prosttico, por lo que la intervenci n quirrgica no protege del desarrollo tivas de la masa, sumando ambas puntuaciones para obt ener un res ultado
de este proceso. final de 2 a 1O. Esta escala de Gleason se corresponde con el pronstico de
la enfermedad, independientemente del estadio. Pa ra la estadificacin se
Los tratamientos no quirrgicos son fitoterapia (poco efectivo); antagonis- emplea principalmente la clasificacin TNM (Tabla 11 y Figura 17).
tas a-adrenrgicos (tamsulosina, silodosina, doxazosina, etc.) que relajan la
musculatura del cuello vesical y de la uretra; inhi bidores de la 5-a-reductasa
(finasteride, dutasteride) reducen el tamao prosttico. En casos avanzados
co n prstata mayor de 40 cm 3 se recomienda iniciar con te rapia combina-
da de estos frmacos. Para los sntomas de llenado adems se pueden aso- Tl: tumor inaparente clnicamente (no palpable ni visible
ciar antimuscarnicos (sol ifenacina, oxibuti ninia, tolterodina ... ) o agonista por tcnicas de imagen)
a-3 (mirabeg ron) que son f rmacos utilizados para vejiga hiperactiva . Tl a: hallado incidentalmente. Afectac in menor del 5% del tejido
resecado
- T1 b: hallado incidentalmente. Afectacin mayor del So/o del tejido
resecado
- T1 e: Tumor identificado por puncin -bi opsia por aumento del PSA
La finaster ida tambin es til para la alopec ia andro-
gnica, donde se empl ea en dosis mucho menores. T2: tumor confinado a la prstata (incluye la invasin de la cpsula
Recuerda prosttica sin exteriorizacin del tumor hacia el tejido adiposo
periprosttico)
- T2a: menos del 50% de un lbulo
Como inconvenientes principales de los in hibidores de la Sa-reductasa, T2b: ms del 50% de un lbulo
- T2c: dos lbulos
se encuentran: impotencia, reduccin del PSA en torno aISO% (dificultan-
do el diagnstico del carcinoma, si lo hubiese) y que tarda una media de T3: extensin del tumor por fuera de la cpsula
cuatro meses en hacer efecto. - T3a: extensin transcapsular (sea unilateral o bilateral)
- T3b: invas in de la (s) vescula(s) sem inal(es)

De los a-bloqueantes los inconvenientes principales son la hipotensin


T 4: tumor fijo o invade rganos adyacentes distintos a las vesculas
(ms frecuente en los frmacos antiguos) y la eyaculacin retrgrada. seminales (cuello vesica l, esfnter externo, recto, msculo elevador
o pared pelviana)
En cuanto a las indicaciones de tratamiento quirrgico, globalmente, slo un N: define la afectacin ganglionar
1O% de los pacientes prostticos precisar ciruga. La intensidad de las mani- NX: no se pueden estudiar los ganglios regionales
festaciones clnicas subjetivas y la mala respuesta al tratamiento mdico pue- NO: no metstasis ganglionares
den constitu ir la indicacin para la intervencin. Entre las causas "objetivas" Nl: metstasis a ganglios regionales
que suponen indicacin absoluta de tratamiento quirrgico se encuentra n: M: define las metstasis
Retencin uri naria reiterada. MO: no metstasis
Hidronefrosis retrgrada (lesin del parnquima renal por obstruc- M1: metstasis a distancia
cin infravesical). M 1a: ganglios linfticos no regionales
Infeccin urinaria de repeticin . - Ml b: hueso
- M 1e: otras localizaciones
Litiasis vesical.
Hematuria de repeticin. Tabla 11 . Estadificacin del carc inoma de prstata
Manual CTO de Medicina y Ciruga, 2.a edicin

T: tumor primario clnica TN

T2a T2b
TX No puede eva luar el tumor
TO No existen signos de tumor primario

Tla Tlb

< So/o > So/o


T2 Tumor lim itado a la prstata T3 Tumor que se extiende a travs
Tl Tumor no evidente clnicamente, no pa lpable de la cpsula prosttica:
o a la cpsula, sin sobrepasarla:
ni visible mediante tcnicas de imagen: T3a Extensin extracapsular (unilateral
T2a Menos del SO% de un lbulo
Tl a Extensin menor o igual al So/o del tejido o bilateral)
T2b Ms del SO% de un lbulo
resecado T3b Tumor invade la vescula seminal
T2c Dos lbulos
Tl b Extensin mayor del So/o del tejido
resecado
. Tl e Tumor identificado mediante puncin
bipsica (consecuencia de un PSA elevado)

T4 Tumor fijo o que invade estructuras adyacentes diferentes a las vescu la s sem inales NX No se pueden eva luar los ganglios linfticos regionales
NO No hay metstasis ganglionares regionales
Nl Metstasis en ganglios linfticos regional es

Figura 17. Est adificacin del adenocarcinoma de prstata

screening para tratar de aproximar estas cifras a las de EE.U U. o Europa,


Clnica donde ms dei ?Oo/o de estos tumores se d iagnostican en fases temprana s
de la enfermedad.
El carcinoma prosttico es una enfermedad ms frecuente en ancianos,
y la mayora de ellos se diagnostica por encima de los 60 aos. Clnica- Marcadores tumorales
mente puede producir sntomas obstructivos del tracto urinario inferior
superponibles a los de la HPB. A est os puede aadirse la hematuria. El Se dispone fundamenta lmente de dos marcadores tumorales. La fosfa-
25% de los pacientes que refieren reten cin urinaria aguda presentan un tasa cida prosttica (FAP) se emplea en clnica desde hace dcadas, es
carci noma prosttico. Aproximadamente un 25% de los pacientes pre- un marcador especfico, pero su elevacin suele indicar extensin extra-
sentan metstasis en el momento del diagnstico; est as pueden produ- prosttica, por lo q ue no resu lta til en el d iagnstico precoz. El antgeno
cir manifestaciones como dolor seo, compres in medular, mielopti sis prosttico especfico (PSA) es realmente un marcador de tejido prosttico
o coagu lopata. Afortunadamente, est os casos se encuentran en claro cuyos nive les suelen encontrarse ms elevados en el cncer, pero es ines-
descenso gracias a la incorporacin de l PSA (prostate-specific antigen-an- pecfico y tambin est n elevados a consecuencia de patologa benigna
tgeno prost tico especfico), facilitan do el d iagnstico de la enfermedad (infecc iones, sondajes, HPB, etc.). Por este motivo se ha intentado aumen-
en estadios tempranos y comnmente asintomticos. tar su especificidad para c ncer con otros pa rmetros (densidad de PSA.
ndice PSA/edad, ve locidad de cambio del PSA, PSA li bre), aunque an no
Diagnstico ha quedado establecida la ventaja de estos sobre el PSA aislado.

Tacto rectal Si el PSA es menor de 4 ng/m l, es poco probable que se encuentre un


cncer de prstata. Si es mayor de 1O, las probabilidades aumentan, lo
Contin a siendo el mtodo fundamental de cribado. Son accesibles al que aconsejara una biopsia de prstata ecod irig ida. Si est entre 4 y 1O,
tacto recta l todos los estadios excepto el T1 , que por definicin es un se pueden utilizar los parmetros antes mencionados para valorar la ne-
ha llazgo. Ca ractersticamente, el ca rcinoma es du ro, nodu lar e irregu lar. cesidad de biopsia.
En genera l, se aconseja un tacto rectal y un PSA anua l a todos los va rones
por encima de 50 aos aunque, de momento, la OMS no aconseja la rea - Pruebas de imagen
lizacin de cribado poblacional sistemtico. No obstante, en Mxico en la
actual idad el ?So/o de los cnceres de prstata se diagnostica ya en estado La ecografa transrectal (ETR) (Figura 18) es el mtodo de imagen ms
avanzado, por lo que sera conveniente plantearse la rea lizacin de un til para la estadifi cac in loca l, pudiendo ofrecer informacin importante

05 Hiperplasia y carcinoma prosttico


Urologa 1 05
sobre la afectacin capsular, de vesculas seminales, cuello vesica l o recto.
Aunque no existe un patrn caracterstico, suele aparecer como ndulos
hipoecognicos. La ecog rafa transrecta l ofrece, adems, la posibilidad de
dirigir la biopsia hacia las zonas sospechosas.

La ecografa abdominal no t iene gran valor en la deteccin del carcinoma


prosttico. La TC y la RM t ienen su principal papel en la estadificacin
ganglionar y la va loracin de metstasis a distancia. Las primeras mets-
tasis deben bu sca rse a nivel de los ga nglios linfticos de la s ca denas ob -
tu ratrices e il a cas.

Figura 19. Radiografa de columna . Metstasis osteoblsticas

Las metstasis del c ncer de prstata son osteob lsti-


cas, es decir, form an hueso (se ven mejor en la radio-
grafa simp le, no en la gammagrafa) . Recuerda

Biopsia prosttica

Debe realizarse para la co nfirma ci n del diagnstico. Pu ede efectuar-


se va transrectal o transperi neal, guiada por el tacto rectal o bie n
guiada por la ecog rafa tra nsrectal, lo que aade efectividad a la p ru e-
ba. La rea li zaci n de la biopsia est ind icada siempre que ex ista una
anomala del t acto recta l, elevacin de los marcadores tumorales o
alteracin en las pruebas de imagen. La puncin-aspiracin con agu-
ja fina (PAAF) es una alternat iva con menores compl icaciones, pero
con el inconveniente de que no puede evaluar el g rado hi stolgico
(G ieason).

Son indica ciones de biops ia prosttica el tacto recta l


sospechoso, la presencia de un ndulo ecogrfico y
un PSA > 4 (variabl e la c ifra segn crite ri os) .

Tratamiento
(A) ETR corte transversal; (B) ETR corte longitudinal; (C) Adenocarcinoma, Opciones teraputicas
ndulo hipoecoico en lbulo derecho
Figura 18. Ecografa transrectal (ETR) de adenocarcinoma prosttico Prostatect oma radical . Los pacientes candidatos deben ser ind i-
viduos con una espera nza d e vida superior a 1O aos. Como com-
Gammagrafa sea plicaciones, se puede encontrar incontin encia (2-57%), estenosis
anastomtica (10%), impotencia (50%) o incluso la muerte(< 5%).
Se utiliza para la deteccin de metstasis seas, ti ene mayor se nsibi lidad En ln eas genera les, suele ir acompaada de linfadenectoma leo-
que la radio log a convenciona l (Figura 19), y debe rea liza rse en todo obtu ratriz.
paciente en qu ien se sospeche metstasis (Gieason > 8, PSA > 20). An- Radioterapia. Como tratamiento curativo, los resultados en esta-
tes de plantearse el tratamiento curativo, en ciertos pacientes con altas dios loca lizados se acerca n a los de la ciruga. La diarrea crn ica, la
probabil idades de encontrarse el cncer ext end ido, se debe efectu ar una proctitis, la cistitis rd ica y las fstulas urinarias son com plicaciones
gammagrafa previa para confirmar la no existencia de metstasis seas o del tratamiento, as com o la incontinencia y la impotencia a pa rtir de
una TC para desca rtar metstasis ganglionares. los dos aos de tratam iento.
Manual CTO de Medicina y Ciruga, 2.a edicin

Se ha em p leado ta m b in rad ioterapia intersticial (braqu iterapia) cientes j venes (menores de 60 aos) con una elevada esperanza
con implantacin de yodo- 123 (1- 123), oro- 198 (Au- 198), pa lad io de vida.
e irid io. Su indicacin queda lim itada a tumores pequeos de es- Estadio T1 b-Tl c. Alcanzan una mort al idad del 80o/o dejados a su
tadio T1 o T2, y sus resu ltados son simi lares a los de la ciruga. En evolucin natura l. Por ello est indicada la prostatectom a radical, la
caso de comp resin m ed ular o dolor por m et stasis seas, la ra- rad iot erapia extern a o braq uiterapia, en sujetos con espera nza de
di oterapia sobre la met stas is p uede conseguir el co ntrol loca l de vida superior a 1Oaos.
la enfe rm edad. Estadio T2a. Es la indicacin ms cl ara de prostatectoma rad ica l. La
Vigilancia activa: en los tumores de bajo ri esgo, es posible rea lizar radioterap ia o braqu iterapia se reservara pa ra pacient es de riesgo
vigilancia con controles perid icos y en caso de progresin realizar qu irrgico elevado o que no aceptan efectos secunda rios atribuibles
tratamiento con intencin curativa. a la ciruga.
Hormonoterapia. El adenocarcinoma prosttico est compuesto Estadio T2b y T2c. Un 40o/o demuestra ser en rea lidad estadio 3, tras
por una poblacin heterognea de clulas androgenodepend ientes el an lisis de la pieza quirrgica de prostat ectoma radica l (infraesta-
y and rogenoindependientes. La supresin horm onal frena el creci- dificac in). La radioterapia externa o braquiterapia ta mbin puede
miento de las primeras, pero no afecta a las androgenoindependien- ser til en pacientes de alto ri esgo quirrgico.
tes. Se puede conseg uir dismi nu ir los niveles de andrgenos circ ula- Estadio T3a. La ind icacin quirrgica es dudosa, as como la radio-
res por distintos mtodos: terapia local, por lo que solamente se propondra a sujetos jvenes,
Castracin quirrgica. Es el mtodo aislado ms eficiente, aun a costa de obtener malos resulta dos. Genera lment e son tratados
con la ventaja de que elimina la neces idad de medicacin como el grupo siguient e.
permanente. Por su rapidez en el efecto supresor hormona l, Estadio T3b, T4, N+, M+. Va r n ao so con mal estado genera l.
tambin est indicada en las compresiones med ula res por El tratam iento hormona l es la opci n ind ica da. Puede ser preciso
metstasis. el uso de radioterap ia paliativa sobre la metstasis en caso de
Estrgenos (dietilestilbestrol). Inhibe la secrecin de LH. Ac- dolor.
tua lmente este mtodo se ha abandonado debido al alto riesgo
ca rdiovascu lar que conlleva. Recid iva t umoral post erior a tratamiento
Progestgenos. Inhi ben la secrecin de LH y acta n como an- con int encin curat iva
t iandrgenos, al unirse a los receptores de la dihidrotestoste-
rona. Es preciso aadir estrgenos pa ra evita r el fenmeno de
escape, que se produce t ras varios meses de tratam iento. No Tras la rea lizacin de p rostatectoma rad ica l, los pacientes son monitori-
son de uso habitual. zados genera lmente con peticiones de PSA.
Agonistas LHRH. Aunque inicia lmente ocasionan un aumento
de los niveles de testosterona, posteriormente suprimen la se- Cuando las cifras de PSA t ras prostatectoma rad ica l son superiores a 0,4
crecin de LH y de andrgenos. La elevacin transitoria de los ng/m l, se considera recidiva bioqu mica y debe hacer sospechar la exis-
and rgenos puede empeorar el cuad ro cln ico, principalmente tencia de met stasis a dista ncia, o bien la exist encia de rec idiva a esca la
si existe compromiso medu lar por metst asis seas. Esta eleva- loca l.
cin (flare-up) se debe suprim ir media nte la administracin de
antiandrgenos, previamente a la introduccin de inhi bidor de Tras la rea lizacin de rad iot erapia como t rata m iento de cncer de prs-
la LHRH. tata loca lizado, los descensos de PSA va n siendo pa ulatinos (a diferencia
Antiandrgenos (bicalutamida, flutamida, acetato de ci- de la prost at ectoma rad ica l) hasta conseguir un valor nadir, que es el va-
proterona). Compiten con el recept or androgn ico. Suelen lor mni mo alca nzado t ras el t rata mient o y que se considerar referencia
uti lizarse con agon istas de la LHRH. El acetato de ciproterona, para el seguimiento posterior.
adems de actua r como antiandrgeno, t iene un efecto pro-
gestgeno, por lo que acta a nivel central, disminuyendo los Existen diferentes criterios para considerar el diagnstico de recidiva
pu lsos de LH. bioqum ica tras tratam iento con rad ioterapia: cuando se constata la exis-
Quimioterapia. No es muy efectiva en el adenocarcinoma tencia de t res elevaciones sucesivas a partir del va lor nadi r, cuando se
prosttico. Se han rea lizado trat amientos con f rm acos, que evidencian niveles nadir+2 (criterio de la ASCO), o con valores nad ir+3
son una mezcla de un estrgeno y una mostaza nitrogenada (criterio de Phil adelphia).
(fosfato de estramustina), pero los estudios son contrad ictorios
y las respuestas pobres. Existen estudios prometedores con el Tratamient o de urgencia
empleo de docetaxel en pacientes con tumores hormonorre-
sistentes. La compresin medu lar por el cncer prosttico no tratado puede ser la
forma de presentacin y constituye una urgencia importa nte. El objeti-
Otros: se estn diseando nuevas molcu las que bloq uean los an- vo del tratam ient o debe ser la supresin andrognica rpida o la des-
drgenos a distintos niveles como la abiraterona y la enza lutam ida, compresin medu lar m ed iante laminectoma qu irrg ica o radioterapia.
utilizndose cuando el tumor se hace resistente a la hormonoterapia Se pueden dism inui r los niveles de andrgenos med iante castracin
convencional. quirrgica urgente, ketoconazol en altas dosis o d iet ilestil bestrol intra-
venoso.
Trat amiento por estadios
A continuacin se expone una tabla que resume t anto las caract ersticas
Estadio T1 a. Tienen una mortal idad por la enfermedad del 2o/o a de la hiperplasia prosttica benigna (HPB) como las del adenoca rcinoma
los 1Oaos, por lo que no precisan tratamiento, salvo quiz los pa- prosttico (Tabla 12).

05 Hiperplasia y carcinoma prosttico


Urologa 1 05
Adenocarcinoma prosttico
Localizacin Zona transicional Perifrica
Clnica Fases: Mayora asintomticos:
Compensacin Hasta 25% Sndrome prosttico
Clnica Hasta 25% Retencin aguda
Descompensacin Hasta 25% Metstasis
Diagnstico Tacto rectal
diferencial Ecografa transrectal (estad ificacin local)
Gammagrafa osea (metstasis seas)
PSA (muy sensible, poco especfico). Descarta cncer prosttico, pero no diagnostica HPB
Fosfatasa cida (muy especfica, poco sensible)
Biopsia (con firmacin)
Tratamiento Fitoterapia Localizado: prostatectoma radical ms linfadenectoma
Frmacos: finasterida, a-bloqueadores bilateral, radioterapia
Ciruga: adenomectoma: endoscpica o abierta Ava nzado: castracin: quirrgica (eleccin), farmacolgica

Tabla 12. Tabla-resumen de las caractersticas de la HPB y del adenocarcinoma prosttico

" El PSA elevado no es diagnstico de cncer de prstata. Pue-


Ideas clave RS de corresponder a una HPB. El diagnstico definitivo de cncer
prosttico precisa una biopsia.
" La hiperplasia prosttica benigna (HPB) suele afectar a la zona
periuretral de la glndula. El cncer aparece en la zona perifrica. " Las metstasis lumbares son tpicas del cncer de prstata, pu-
diendo producir compresin medular.
" La hiperplasia prosttica benigna no guarda relacin con el cncer.
" El tacto rectal revela una prstata ptrea e irregular en el cncer
" Tanto la HPB como el cncer tienen relacin con las hormonas de prstata. Sin embargo, al principio puede no ser palpable, ni
sexuales, y suelen aparecer en varones ancianos. visible en la ecografa (Tl ).

" El tratamiento mdico de la HPB consiste en a-bloqueadores (re- " La principal complicacin quirrgica del cncer de prstata es
lajan la musculatura uretral y del cuello vesical), inhibidores de la la impotencia.
5a-reductasa (disminuye el tamao glandular) y fitoterapia. Esta
ltima no ha demostrado utilidad con parmetros objetivos. " Ante un sndrome de compresin medular por cncer de prs-
tata, nunca se deben emplear anlogos de la LHRH nicamente.
" El tratamiento definitivo de la HPB es la ciruga, que puede con- Siempre deben asociarse antiandrgenos.
sistir en reseccin transuretral o en ciruga abierta, dependien-
do del tamao prosttico. " En el cncer de prstata, la indicacin ms clara de prostatecto-
ma radical es el estadio T2a.
" El cncer de prstata es casi siempre un adenocarcinoma, con
gran frecuencia multifocal. " El tratamiento fundamental del cncer de prstata diseminado
es la hormonoterapia.
" El cncer de prstata cada vez se diagnostica con ms frecuen-
cia en fase asintomtica. Cuando presenta clnica, puede consis-
tir en sntomas urinarios similares a la HPB.

4) La utilizacin de bloqueo hormonal en este caso no es una op-


Casos clnicos cin de tratamiento posible.

Paciente de 66 aos, intervenido de prostatectoma radical, RC: 3


hace 3 aos por adenocarcinoma de prstata Gleason 8 (pT2b
NOMO). Presenta, en el momento actual, una cifra de PSA srico Hombre de 77 aos que refiere clnica de prostatismo de aos
de 12 ng/ml. Seale cul de las siguientes afirmaciones le pare- de evolucin, y que presenta elevacin del PSA (antgeno
ce correcta: prosttico especfico) (89 ng/ml} y dolor en columna lumbar
desde hace 2 meses. Al tacto rectal, la prstata est aumen-
1) La supervivencia media en el momento actual es menor de 1 tada de tamao, de consistencia dura en ambos lbulos, su-
ao. perficie nodular y lmites mal definidos. Tras realizarle una
2) La cifra de PSA est en rango normal, ya que existen otras fuen- ecografa transrectal con biopsias prostticas ecodirigidas, es
tes de produccin del mismo. diagnosticado de un adenocarcinoma de prstata pobremen-
3) El paciente puede tener una recidiva loca l o bien metstasis a te diferenciado, que afecta a ambos lbulos y que infiltra las
distancia. vesculas seminales. La gammagrafa sea confirma la presen-
Manual CTO de Medicina y Ciruga, 2.a edicin

cia de metstasis en columna lumbar. Qu tratamiento de los A la consulta acude un paciente de 54 aos con molestias a la
siguientes aconsejara en primer lugar? miccin. Refiere disminucin del chorro, dificultad para el inicio,
goteo posmiccional, sensacin de tenesmo y nictu ria de tres ve-
1) Prostatectoma radical. ces. Presenta cult ivos negativos y PSA de 2, l. En la ecografa ab-
2) Quimioterapia intensiva. dominal se objetiva una glndula prosttica de 43 cm 3 Al tact o
3) Hormonotera p ia. rectal no se palpan ndulos sospechosos. En eiiPSS obtiene una
4) Radioterapi a pelviana extern a. puntuacin que permite clasificar su sintomatolog a de modera-
da-grave. Su actitud deber ser:
RC: 3
1) Debido a su edad, el primer paso ser iniciar tratamiento con
Un paciente de 67 aos acude a Urgencias por presentar en los fitoterap ia.
ltimos das debilidad progresiva de miembros inferiores, difi- 2) Debido a la gravedad de los sntomas se debe plantear ciruga
cultad miccional e incontinencia fecal. En la exploracin fsica de entrada.
destaca cierta hipotona anal, con una prstata muy sugerent e 3) Se debe iniciar tratamiento con a-bloqueantes.
de malignidad al tacto, y debilidad de extremidades, conservan- 4) La mejor opcin ser iniciar tratamiento combinado con
do la sensibilidad tctil. Con el probable diagnstico de carci- a -bloqueantes+ inhibidores de la Sa-reductasa.
noma de prstata metastsico, cul de las siguientes opciones
considera MENOS indicada para el tratamiento de urgencia? RC: 4

1) Estrgenos intravenosos.
2) Anl ogos LHRH.
3) Ketoconazol (a ltas dosis).
4) Rad ioterapia.

RC: 2

sided nodule. An ultrasound-guided biopsy is performed and t he


Case Study . result is reported as prostate adenocarcinoma with a Gleason st ore
of 8 (4 + 4). Bone gammagraphy shows t wo lesions that probably
A 63-year-old man is brought to your institution with complaint s correspond to met ast atic lesions in the lumbar spine and scapula.
of voiding problems. He claims he has difficulty in st arting mict u- Mark the incorrect sent ence about prostate cancer met ast at ic le-
rition, weak voiding and postvoid dribbling. He presents nicturia sions:
twice per night. Laboratory test revea ls PSA: 1.37 ng/ml. Urolo-
gic ultrasound study shows normal kidney and urinary bladder 1) The vertebrae, sternum, pelvis, ribs and fe mur are the most
with a prostate that measures 45 ce. common ly affected bones.
2) Osteoblastic and osteoclastic activity increase, but osteoclas-
1) Th is pat ient does not need med ica ! treatment right now. t ic activity is higher and hence osteolytic pattern is more fre-
2) A prostate biopsy should be performed. quently observed in radiographic studies.
3) a.-blocker agents should be prescribed. 3) Pain, inflammation and pathologic fractures are frequent clini-
4) Finasterid e treatm ent is ind icated. cal manifestations.
4) Castration improves symptoms in up to 80% of patients with
Correct answer: 3 metastatic prostate cancer.

A 63-year-old patient comes to the physician beca use he is found Correct answer: 2
to have a PSA of 5.3 on a routine blood analysis. He claims he
has no urinary symptoms or sexual dysfunction. In this particular A 71-year-old maJe who has been found to have a PSA of 16 un-
case, which of the following options is correct? dergoes a t ransrectal biopsy that suggest s a prostate cancer with
a Gleason score of 7 (4 + 3). 1nitialgammagraphy and abdomino-
1) The next step in t he diagnostic workup shoul d be a transrecta l pelvic CT sean do not show any suspicious lesions. The pat ient
ultrasound study. receives a radical prostatectomy. Five years later he complains
2) Th e symptoms are not consisten! w ith prostate cancer. of right scapula pain and laboratory test reveals PSA 6.02 ng/ml.
3) The likelihood of having prostate can cer is approximately 25% A new bone gammagraphy shows three suspicious lesions that
given his PSA level. suggest metastatic lesions, and two of them are localized in the
4) The next step should include a radical prostatectomy. right scapula. In this case, which of the following sent ences des-
cribes the best reaction ?
Correct answer: 3
1) Radiotherapy of the metastatic lesions is ind icated .
A 71-year-old patient presents to his physician with the results of a 2) Com plete hormone-blocking t reatment should be in itiated.
blood test showing PSA levels of 15.3 ng/ml. He says he has some 3) A wa it-and-see attitude is reco mmended, if no pathologic frac-
voiding problems, with frequent interruptions and dribbling. He tures have occurred .
often wakes up at night to urinate. He says he has no hematuria, 4) Curative rad iotherapy shou ld be admin istered.
incontinence or sexual impairment. Digital rectal examination re-
veals an adenomatous prostate grade 111/IV with a suspicious left- Correct answer: 2

05 Hiperplasia y carcinoma prosttico


_UrDlo_gu........a_ _


Carcinom as del tracto ur1nar1o

ORIENTACIN El adenocarcinoma primario vesical es un tumor raro, aunque es el que se


Es fundamental/a parte
ENARM de tratamiento. ha visto asociado a la extrofia vesica l con mayor frecuencia.

No hay que confundir Schistosoma haematobium


con Schistosoma mansoni, que produce hipertensin
6.1. Carcinoma vesical porta l.

El carcinoma vesical es la segunda neoplasia urolgica en frecuencia . Histologa e historia natural


Aparece ms frecuentemente en varones (2-3:1) y ms en poblacin
blanca que negra . Su edad de mxima incidencia se sita entre los 60 y Haciendo referencia al carcinoma de clu las transicionales, hay que di-
70 aos. De ellos, el 90% son carcinomas transicionales, el 8% escamosos ferenciar tres formas de la enfermedad con comportamiento, prons-
y el resto adenocarcinomas. El epitelio urotelial recubre el tracto urinario tico y tratamiento completamente distinto (Tabla 14 y Figura 20). El
desde las papilas caliciales hasta la uretra prosttica, ambas inclusive. En 70% de los tumores vesica les se presentan como tumores papilares de
cualquiera de estos niveles pueden desarrollarse los tumores urotel iales, crecimiento principalmente endocavitario y frente de invas in nico.
correspondiendo la mayora a la vejiga (ms del 90%) y, ms raramente, al Un 10% son slidos, con invasin tentacular en profundidad y exten-
tracto urinario superior (S%) o la uretra (1 %). sin linftica y vasc ular temprana. El 20% restante son formas mixtas.
El primer grupo suele corresponder a tumores superficiales de bajo
Entre los factores etiolgicos (Tabla 13), se implican las aminas arom- grado histolgico, mientras que los slidos, con mayor frecuencia, son
ticas, presentes en las industrias textiles, qumicas y del caucho. El humo tumores infiltrantes de grado histolgico ms elevado. La principal
del tabaco es el principal factor de riesgo (50-60% aparecen en fuma- caracterstica de los tumores papilares superficia les es la recurrencia ,
dores), aumentando el riesgo a mayor consumo. Tambin pueden jugar que ocurre en un 50-75%, segn el grado y estad io. El 25% recurrirn
un papel importante los edulcorantes artificiales (sacarina, ciclamato), la y progresarn en grado y estad io, y nicamente el 15% acabar desa-
ciclofosfamida, los acetiladores lentos (mayor riesg o) y muchas otras po- rrollando un tumor infiltrante o metastsico. La mayora de los tumo-
sibles etiologas. La infestacin por Schistosoma haematobium aumenta res infiltrantes se encuentran confinados a la vejiga en el momento
la incidencia de carcinoma escamoso vesical, as como la presencia de del diagnstico, y slo un 20-25% presentan extensin ganglionar o
infeccin crnica o catter vesical permanente. metastsica. El 50% desarrollarn metstasis a distancia, a pesar del
tratamiento.

Ami nas aromticas (2-nahilamina): tabaco, Transicionales (90%) El tercer grupo que merece mencin aparte es el carcinoma in situ. A pe-
industria textil, industria del caucho, colorantes Mejor pronstico sar de encontrarse lim itado al urotelio, por lo que es superficial, est for-
Fenacetinas crnicas mado por clulas poco diferenciadas con displasia grave. Tiene una alta
Sacarina, cicla mato tasa de recidiva y progresa hacia tumor infiltrante en el 50-75% de los ca-
Ciclofosfamida (acrolena) sos. Este mal pronstico le confiere un ca rcter completamente distinto
Tabaco: ortofenoles, triptfano del carcinoma in situ de otras regiones, en las que se considera el estadio
Schistosoma haematobium Es ca mosos (8%) inicial de la enfermedad tumoral.
Litiasis, infecciones, catteres
El carcinoma in situ puede estar asociado a focos de carcinoma su-
Cistitis glandular Adenocarcinomas (2%)
perficial (26%) o infiltrante (60%) o bien encontrarse de forma aislada,
Extrofia vesical
siendo generalmente mu ltifocal tanto en vejiga como en otros puntos
Tabla 13. Factores etiolgicos de los carcinomas del tracto urinario del urotelio.
Manual CTO de Medicina y Ciruga, 2.a edicin

de microhematuria asintomtica, descubierta durante estudios de criba-


T: define el tumor do, slo se relaciona con enfermedad significativa en menos del 2o/o de

Tis: carcinoma in si tu (plano) los casos. Pueden encontrarse sntomas irritativos (escozor, polaquiuria,

Ta: carcinoma papi lar no infi lt rante tenesmo) en el 2S-30o/o, solos o acompaando a la hematuria. La presen-
cia de un sndrome cisttico no justificado por infeccin o litiasis debe
Tl: tumor que invade tej ido co njuntivo subepitelial
hacer sospecha r la presencia de un carcinoma vesical, especialmente por
T2: tumor que invade msculo
su asociacin con el carcinoma in situ. Con menor frecuencia, el pacien-
- T2a: tumor que invade la mitad interna
te consu lta por dolor en fla nco por obstruccin uretera l, plvico o por
- T2b: tumor que invade la mitad externa
edema en m iembros inferiores (extensin linftica). La exploracin fsica
T3: tumor que invade tej ido perivesica l: suele ser irre levante, sa lvo en la enfermedad avanzada .
- T3a: microscpicamente
- T3b: macroscpicamente (masa extravesical) Las citologas urinarias son una prueba sencilla y fiab le que debe ser reali-
T4a: tumor que invade prstata, tero o vagina zada en todos los casos de hematuria asintomtica o sospecha de tumor
T4b: tumor que invade pared plvica o pared abdom inal vesica l. Su sensibilidad depende del grado de diferenciacin del tumor
N: define la afectacin ganglionar vesica l, alcanza ndo el 7S- 1OOo/o en tumores de alto grado y carcinoma
Nx: metstasis gangliona r regiona l desconocida in situ, siendo en este ltimo un mtodo d iag nstico ms rentable que
NO: ausencia de metstasis ganglionar regiona l la ecografa, la TC la urografa o la biopsia mltiple. Son especia lmente

Nl: metstasis a un solo ganglio entre 2 y S cm ti les en el seguim iento de pacientes sometidos a resecc in transuretral
en combinac in con la cistoscopia.
N2: metstasis en un ganglio mayor de S cm o m ltiples no mayores
de S cm
N3: metstasis mayores de S cm Entre las pruebas rad io lg icas destacan la ecografa (con una sensibilidad
del 80o/o, pero poco til para el d iagnstico de las neoplasias del tracto
M: define las metstasis
urinario superior [Figura 21]) y la urografa intravenosa (capaz de detec-
MO: no metstasis
tar la presencia de t umor en el 60o/o de los casos).
M 1: metstasis a distancia
Tabla 14. Estad ificacin del carcinoma vesica l

Figura 21. Ecografa. Tumor vesica l con zonas calcificadas


1. Mucosa ------i~ Ta, Tis 4. Grasa ----~T3

2. Submucosa - -- Tl S. rganos vecinos - T4 En la urografa intravenosa, adems de la presencia de defectos de


3. Muscular T2
rep lecin, se puede sugerir el diagnstico de tumor vesical, rigidez y
Figura 20. Esquema de la estadificacin del tumor vesica l falta de distensibi lidad vesical, la obstruccin de un urter o el despla-
zamiento de la vejiga, entre otros. En caso de dudas sobre el tracto uri-
Se puede encontrar en el mbito vesical d istintas lesiones ben ignas que nario superior, se recurrir a la pielografa retrgrada en el momento de
no se asocian con el desa rro llo de cncer: los nidos de Von Brunn, la cis- rea lizar la cistoscopia. Esta es fundamenta l para la eva luacin del tumor
titis qustica y glandular orig inados en procesos inflamatorios o irritativos vesica l; puede rea lizarse bajo anestesia loca l cuando existan dudas con
crnicos, y que probablemente sean d istintas man ifestac iones de un mis- las pruebas real izadas previamente, pero si el d iagnstico de presun-
mo proceso, aunque pueden plantea r el diagnstico diferencial; (ocasio- cin es firme, y dado que en todo tumor debe rea lizarse reseccin tran-
na lmente se ha n descrito adenoca rcinomas vesica les asociados a la cisti- suretra l para eva luar el grado de infi lt racin, se puede esperar a tener
t is g land ular). Otras lesio nes benig nas seran el adenoma nefrognico, el al paciente en q ui rfa no baj o anestesia genera l o raqudea pa ra practi-
pli po sim ple, el papilo ma invertido o el papiloma ve lloso. car la. En un 1Oo/o de los casos de cistoscopia co n cito log as posit ivas no
se encuent ra t umor en la vejiga, lo q ue puede ser debido a la presencia
Diagnstico de ca rcinoma in situ, t umor en vas urinarias altas, a un carcinoma duc-
ta l de prstata o a un fa lso positivo de la prueba (genera lmente por
La hematuria macroscpica o microscpica monosintomtica es el ha- inflamacin de la pared vesica l o por tratamiento concomitante con
llazgo ms frecuente, presente en el ?So/o de los pacientes. La presencia radioterapia o quimioterapia endovesical).

06 Carcinomas del tracto urinario


Urologa 1 06
La TC se utiliza para la estadificacin de la enfermedad infiltrante, ya que dir la nefropata por abuso de analgsicos (fenacetinas) y la nefropata de
aporta informacin limita da sobre la infiltracin tumoral. La RM consigue los Balcanes.
mejores imgenes de la cpu la vesica l por sus cortes sag itales, pero no
aporta mayor informacin que la TC. En el 70-80% de los pacientes aparece hematuria macroscpica, siendo
el dolor c lico por obstru ccin ureteral la seg unda queja en frec uencia
Tratamiento de aparicin.

Todo el proceso diagnstico va encaminado a establecer si el tumor vesi-


cal es superficial o infiltrante, ya que el tratamiento vara radicalmente en
funcin de este hecho. La hematuria tambin es la manifestacin ms fre-
cuente del hipernefroma .

Los tumores superficiales son manejados mediante reseccin transure.-


tral. Dada la alta frecuencia de recidivas, la mayora se tratan posterior-
mente con instilaciones endovesica les (q uimioterapia o inm unoterapia
loca l) que dismi nuya n la aparicin de nuevos t umores. Entre los qui mio- Diagnstico
terpicos empleados se encuentran la mitom icina, la t iot epa, la adriami-
cina o la epirrub icina. Con todos ellos se consigue red ucir las recidivas La sistemtica diagnstica es bsicamente la misma que pa ra el tumor
alrededor de un 20%. vesical. Es decir, ecografa (detectar hidronefrosis del lado del tumor),
UIV (se ver un defecto de replecin no compatible con un clculo o una
La inmunoterapia endovesical con BCG (bacilo Ca lmette-Guerin) es, sin anulacin funcional de ese sistema excretor), y TC para la estadificacin
duda, la ms eficaz, disminuyendo el porcentaje de recidivas en un 40%, (Figura 22).
siendo tambin tratamiento de eleccin en el carcinoma in situ, con una
eficacia del 70%. Sin em ba rgo, no se util iza en los tumores vesicales como
primera lnea, rese rv ndose pa ra los tumores de riesgo o con m ltiples
recidivas, ya que t ambin se aco mpa a de mayor nmero de efectos se-
cundarios y de comp licaciones, entre los que se encuentran cistitis febril,
sndrome pseudog ripa l, y las ms graves de sepsis, p rostatitis gran uloma-
e incluso muerte. Estos cuad ros precisan tra tamiento
tosa, neumon itis
tuberculosttico completo al menos de tres a seis meses.

En la enfermedad invasora o infiltrante, tras la reseccin transuretra l


para evaluar la afectacin parietal, el tratam iento de eleccin es la
cistectoma radical. La qu imioterapia sistmica, adyuvante o neoad-
yuvante, no pa rece mejora r los res u ltados de la ciruga ais lada . Igua l-
mente se p uede decir d e la rad iote rapia, que nicame nte ha demos-
trado incrementar el t ie mpo li b re de recidiva loca l, sin au mentar la
supervivencia.

La quim ioterapia co mb inada con M-VAC (metotrexat o, vinblastina,


adriam icina, cisplatino) se re serva para pacientes en los que existe
afectacin gang liona r o m etastsica, con respuestas completas en el
15-30% y parcia les en el 30-40%. Tras la cistectoma, los urteres son
derivados genera lmente a segmentos intestinales o a p iel, pud iendo
rea lizarse estomas no contine ntes o rese rvo ri os continentes directa-
mente al remanente uretra l.

La adriam icina (doxorru bici na) es un quim ioterpico


que puede producir cardiotoxicidad.

6.2. Tumores del tracto urinario


superior

Entre el 2-So/o de los t umores uroteliales se encuentran localizados entre (A) Urografa intravenosa con defecto de replecin en pelvis rena l derecha
los cl ices y los orific ios ureterales. En su etiologa estn implicados los (B) TC en fase excretora con tumoracin pilica derecha

mismos factores que para el carcinoma vesical, a los que habra que aa- Figura 22. Ecografa. Tumor vesical con zonas calcificadas
Manual CTO de Medicina y Ciruga, 2.a edicin

En estos tumores, la citologa urinaria aumenta su eficacia si se obtiene de


forma selectiva, cateteriza ndo el urter del lado afectado. Ideas clave ~
" El carcinoma vesical ms frecuente es el urotelial, siendo el ta-
Otros elementos de diag nstico son las biopsias por cepillado y la ure-
baco el principal factor de riesgo.
te ropieloscopia, elemento que, adems de diag nsti co, pu ede tener un
fin teraputico en tumores de peq ueo tam ao y aspecto su perficial " El carcinoma escamoso se relaciona con la esquistosom iasis
(Figura 23). (S. haematobium).

" El adenocarcinoma ves ica l se re laciona con el antecedente de


extrofia vesical.
Fumador+ Hematuria

" El carcinoma papilar superficial y el carcinoma in situ (CIS) son


Sospecha de tumor urotelial muy recurrentes.

" Clnica ms frecuente del carcinoma urotelial: hematuria, ms


f
Citologas+ Pru ebas de im agen
<ECOrvp
tpico con cogulos.
UIV

" Cuando se trata de un carcinoma in situ: sntomas irritativos


(polaquiuria, disuria, tenesmo, etctera).
No concl uyente Concluyente
para tumor vesica l para tumor vesica l
" El mejor mtodo para la estadificacin local es la reseccin
t t
Cistoscopia Ea - - -- - -- - - - RTUV
transuretral.
~-------,

e+
Biopsia vesica l Ea _ _ _
t
__
t t " Prueba ms sensible para el diagnstico de CIS: citologa urinaria.
_____,~T is 2: T2 TA, Tl

e+ + " Conducta ante un CIS: tratar con bacilo Calmette-Guerin (BCG)


y revisiones (cistoscopia y cito logas).
Sospecha r tumor BCG Cistectoma Qu imioBCG (G3)
Tracto urin ari o superior

Re~diva
Ureterorrenoscopia Revisiones
Gstoscopia " Actitud ante un tumor superficial: reseccin transuretral. Pos-
Pielografa retrg rada
Citologas selectivas +Citologla Reci:iva TisLctomfa teriormente, revisiones (cistoscopia y citologas).
Cepillado uretera l
" Actuacin ante un tumor infiltrante (afectacin capa muscu-
lar): cistectoma.
Figura 23. Algoritmo diagnstico-teraputico en tumor de vejiga

Un paciente de 64 aos acude a consulta por sndrome miccional


Casos clnicos de dos meses de evolucin. Refiere que su MAP le ha dado trata-
miento antibitico y le ha realizado cultivos que han sido negati-
Varn de 57 aos, fumador, que consulta por hematuria termi- vos. Se le realiza una ecografa y una flujometra que son normales
nal, polaquiuria, urgencia y dolor miccional. Presenta citologas y una urografa intravenosa que no evidencia alteraciones. En la
urinarias positivas de carcinoma urotelial, y el estudio anatomo- cistoscopia no se observan lesiones intravesicales. Las citologas
patolgico tras la reseccin transuretral es de carcinoma in situ urinarias son sugestivas de malignidad. Seale la incorrecta:
difuso, con intensa inflamacin crnica. El tratamiento estndar
ser: 1) El tratamiento de eleccin ser la inmunomodul acin vesical.
2) Es imprescindible la realizacin de biopsias para confirmacin
1) Instalacin del bacilo de Calmette-Guerin. del diagnstico.
2) Cistectoma radical. 3) Si recidiva tras las instil aciones, se deben repetir los ciclos dos
3) Instilaciones con mitomicina. veces ms.
4) Quimioterapia con cisplatino. 4) La cistectoma es el tratamiento final en muchos de estos pa-
cientes.
RC: 1
RC: 3
Mujer de 63 aos que es diagnosticada de carcinoma de clulas
escamosas del trgono vesical, con invasin de la capa muscular. Una paciente de 58 aos acude por polaquiuria de dos aos de
Cul sera su actitud teraputica en este caso? evolucin, nicturia y dolor hipogstrico. Niega haber visto san-
gre en la orina y es fumadora. Seale la respuesta correcta:
1) Quimioterapia adyuvante, seguida de cistectoma radical.
2) Reseccin transuretral, seguida de inmunoterapia intravesical 1) Se deben realizar biopsias a esta paciente.
(BCG). 2) Una cistoscopia permitir confirmar el diagnstico.
3) Cistectoma radical con extirpacin de cara anterior de vagina. 3) Los tratam ientos empleados hasta el momento han demostra-
4) Rad ioterapia externa, seguida de quimioterapia con cispla- do una gran eficacia .
tino. 4) Muy probablemente los cultivos sern positivos.

RC: 3 RC: 1

06 Carcinomas del tracto urinario


Urologa 1 06
3) A urine PCR test shou ld be performed.
Case Study . 4) Urologic echography is recommended.

A 56-year-old male presents t o his physician with complaint s Correct answer: 2


of self-limit ed hemat uria with no accompanying symptoms. He
says he had a similar episode six weeks ago. He has been smo- A pat ient who underwent a t ransurethral resection of a urinary
king more than one pack a da y for 35 years. He has hypertension bladder lesion has been subsequent ly prescribed BCG immu-
and is currently seeing a neurolog ist beca use he has severe hea- not herapy. Which of th e following is the incorrect answer?
daches. Regarding t his pat ient's diagnost ic w orkup, w hich of t he
following clauses is correct? 1) BCG immunotherapy produces only local symptoms that are
very well tolerated .
1) lf he has a urothelia l tumor, cytology will be always positive. 2) The mechanism of action of BCG consists of an immune-boos-
2) A negative echography rules out a urinary tract tumor. ting effect.
3) A negative cystoscopy rules out the presence of a papillary tu- 3) BCG therapy should not be used in immune-deficient patients
mor in the urinary bladder. such as patients with AIDS.
4) Cytology is only indicated if we have more than one negative 4) BCG therapy should be indicated in patients with intermediate
cystoscopy, in order to ru le out in situ carcinoma. risk tumors and elevated risk of rec urrence or progression .

Correct answer: 3 Correct answer: 1

A 58-year-old female complains of irritat ive micturition syndro- A 65-year-old male undergoes a transurethral resection of a urinary
me for the last few months. Severa! urinary cultures have been bladder t umor. Pathology report describes the lesion as a T2G3.
performed with negative results. She usually smokes, has type Which of the following is t he correct management of this patient?
11 diabetes and hypertension. Urinalysis shows microhemat uria.
Which of the following options is incorrect? 1) BCG therapy should be init iated.
2) Close fo llow-up.
1) lf the cytology is positive, stopping smoking would help dimi- 3) Radical cystoprostatectomy.
nish the recurrences. 4) A complete staging study shou ld be performed .
2) A transurethral resection m ay be necessary in order to stage the
disease. Correct answer: 4
1
Urologa

Tumores testiculares

Este tema es sencillo y rentable siempre que se seleccione lo realmente importante: la clnica
ORIENTACIN y el diagnstico. Es fundamental/a revisin profunda de los Desgloses.
ENARM En cuanto al tratamiento, dado que no existe un protocolo universalmente aceptado, es mejor
conocer ideas generales.

7 .1. Etiologa y epidemiologa Otros factores que se pueden encontrar relacionados son las hernias in-
gu inales infantiles (no demostrado) y la orquitis urliana secundaria al pa-
ramixovirus causante de la parotiditis (siempre que haya producido atro-
Los tumores testiculares suponen el 1-2% de las neoplasias en va rones, fia) y, entre los factores txicos, la exposicin a radiaciones, fuentes de
y son las neoplasias slidas ms frecuentes entre los 20 y 35 aos. Pre- calor, productos para te ido del cuero y estrgenos intrauterino durante
sentan mayor incidencia a mayor desarrollo de la soc iedad, y tambin el primer trimestre del embarazo.
en la raza blanca ms que en las dems. La tasa de cu racin es superior
al90%. Aunque entre un 8 y 25% de los pacientes presentan historia de trauma-
tismo testicu lar, todos los autores pa recen estar de acuerdo en que este
El 95% de ellos proceden de clulas germina les y, aunque globalmente supone ms bien el motivo por el que se descubre una masa escrotal, y
el seminoma es el ms frecuente, la incidencia va ra segn el grupo de no su origen.
edad considerado. El 5% restante se reparte entre tumores del estroma
gonadal (1-2%), linfomas (1 %), gonadoblastomas (clulas germ inales y
del estroma) metstasis y otros. 7 .2. Anatoma patolgica
Los factores de riesgo para el desarrol lo de tumor testicular son criptor-
quidia, historia fami liar de neopla sias te sticu lares en parientes de pri- Los tumores testicu lares se suelen originar como una masa testicular in-
mer grado (padre, hermano), tumor previo en el otro teste, infertilidad traparenquimatosa. A partir de ah, pueden quedarse loca lizados o me-
y sndrome de Klinefelter. tastatizar. La loca lizacin ms frecuente de metstasis son los ganglios
retroper itoneales (casi siempre es el primer esca ln en la diseminacin);
Los testcu los no descend idos tienen mayor ri esgo de desarroll ar tumo- posteriormente puede apa recer la afeccin mediastn ica, de vsceras ab-
res, y esta probabil idad aumenta si la situacin del teste es intraabdom i- domina les y de pu lmn .
na l. Asim ismo, el testcu lo cont ra latera l, aunque de loca lizacin escrota l,
tiene una incidencia mayor; de hecho, el 20% de los tumores en pacien- Hay que tener en cuenta que el testcu lo izqu ierd o drena a los gan-
tes con criptorquidia se desa rrollan en el test culo no criptorqudico. g lios paraarticos y prearticos a nivel de L2. El derecho drena a gan-
Por estas razones, los testculos no descendidos deben descenderse, gl ios interaortocavos, precavos y prearticos, y tambin a nivel del
preferiblemente entre el primer y seg und o ao de edad para fac ilitar su hilio renal. La diseminacin hematgena es m enos frecuente, sa lvo
segu im iento y, con ello, la deteccin de un eventua l t um or. en el coriocarc inoma, v a vasos espe rmticos, siend o los puntos ms
habituales de metstasis: pulmn, hgado, hueso y SNC, por orden de
Una vez alcanzada la pubertad, un teste criptorqud ico no descendido frecuencia.
probablemente deba extirparse, dado que estos testcu los pierden la ca-
pacidad de espermatognesis y conserva n su potencial mal ign izante. No Los tumores testicu lares se dividen en dos grupos: tumores que no de-
obstante, hay autores que prefieren no extirparlos, siempre y cuando se rivan de las clulas germinales (5%) y tumores que derivan de las clulas
puedan descender a la bolsa escrotal, ya que an pueden mantener su germ inales (95%); a su vez, estos ltimos se dividen en sem inomatosos y
funcin endocrina (secretora de testosterona). no sem inomatosos (Tabla 15).
Urologa 1 01
Tumores de clulas germinales Tumores del estroma Tumores de clulas de Leydig. Pueden verse crista loides en su
Tumores de un tipo histolgico Tumores clulas Leydig citoplasma. Generalmente son benignos. Pueden ser hormonal -
Semi noma: Tumores clulas Sertoli mente activos, produciendo pubertad precoz o femin izacin.
Tpico Tumores estructuras Linfoma. Tanto como localizacin secu nd aria o como origen pri-
gonada les primitivas mario (menos frecuente), un a masa testi cular en un va rn mayor
- Anaplsico
- Espermatoctico de 50 aos sugiere, en primer lugar, el d iagnstico de linfoma.
Tras la orqu iectoma o la biopsia testicular para el diagnstico
Carcinoma embrionario de certeza, el tratamiento no vara respecto a los linfomas habi-
Poliembrioma tuales.
Tumor del saco vitelino (seno endodrmico)
Coriocarcinoma
Teratoma: 7 .3. Clnica
- Maduro
Inmaduro
- Con transformacin maligna La manifestacin ms frec uente es como masa escrotal indolora. Con
Tumores de ms de un tipo histolgico mucha menor frecuencia, el motivo de co nsulta est originado por
Teratocarcinoma la presencia de metstasis ganglionares (masas supraclavicu lares o
abdomina les), o efectos endocrinos (g inecomastia, pubertad pre-
Otros
coz). El 1O % de los tumores se presentan como escroto agudo en
Tumores germinales+ estromales
la urgencia.
Gonadoblastoma
Tabla 15. Tu mores de t estcu lo. Clasificacin histolgica

Ex isten algunas enfermedades, como la sarco ido-


Seminoma. Puede aumentar el tamao testicular hasta 1O veces sin sis, que aumentan el tamao testicular sin ex istir
distorsionar su morfologa. Ocasionalmente se presenta extratesticu- un tumor.
lar en mediastino (la ms frecue nte), retroperitoneo, o regin pineal
de forma primaria. Se han descrito tres tipos histolgicos: semi noma
tpico, anaplsico y espermatoctico (ms frecuente en individ uos
mayores de 50 aos). El semi noma p uro no es productor de marca- 7 .4. Diagnstico
dores tumorales, pero hasta en un 15% de los casos pueden aparecer
clulas del sincitiotrofoblasto, produciendo elevaciones de la ~- HCG .
Carcinoma embrionario. Tiende a metastatizar de forma temprana. La exploracin fsica de una masa indolora, sin signos inflamatorios y de
Forma masas ms irregulares y heterogneas que otros tumores. largo periodo de evolucin, debe hacer sospechar un tumor testicular
Tumor del seno endodrmico. Este tumor fue descrito inicia lmen- (vase la Tabla 16).
te como una forma rara de tumor infantil. Se encuentran elementos
de saco viteli na en el 38% de los tumores testiculares del adulto. Un elemento importante, tanto para su diagnstico como para el segui-
Coriocarcinoma. En el estudio histolgico tiene que contener ele- miento, son los marcadores tumorales: a.-fetoprotena (AFP) y fraccin ~
mentos de sincitiotrofoblasto y citotrofoblasto para ser considerado de la gonadotropina corinica humana (~- H CG) (Tabla 17).
como tal.
En el momento del diagnstico, generalmente existen metstasis a La AFP es sintetizada por clulas del saco viteli na y, por tanto, est pre-
distancia (suelen ser va hematgena) y un tumor pri mario testicular sente en tumores de saco vitelina o seno endodermal, y en los carcino-
pequeo. Raramente es puro. mas embrionarios. El semi noma nunca produce AFP En cualquier caso,
Teratoma. Por definicin, se encuentra formado al menos por dos se debe tener en cuenta que la AFP es un marcador inespecfico, y se
capas distintas de clu las germinales (endodermo, mesodermo o podra encontrar elevado en enfermedades hepticas ben ignas o ma-
ectodermo). lignas, algunas neoplasias pancreticas y de la va biliar o en la ataxia-
Teratocarcinoma. Tumor mixto con reas de teratoma y de carcino- telangiectasia.
ma embrionario. Un 64% tiene tambin reas de semi noma.
Gonadoblastoma. Contiene grandes clulas simila res al seminoma
y otras menores, como clul as de Sertoli inmaduras o de la granu lo-
sa. Se asocia preferent emente a las gnadas disgenticas y estados La AFP tamb in se eleva en lqu ido amn itico en al - _.."""'"~"'"
intersexua les. terac iones del cierre del tubo neura l.

Tumores de clulas de Sertoli. Forman reas tubu lares sim ila res a
Recuerda
los t bulos del testculo prepuberal norma l. De comportamiento be-
nigno, es excepcional que metastaticen. La fraccin~ de la HCG es producida por las clulas del sincitiotrofoblasto
presentes en el coriocarcinoma y, tambin de forma ocasiona l y de forma
aislada, en algunos semi nomas.

El tumor de clulas de Sertoli se ha asociado al sn- .,._;;::a.._


1 En conjunto, el 70% de los tumores testiculares producen algn marca-
drome de Peutz-Jeghers
dor, luego existe hasta un 30% de tumores con ma rcadores negativos al
diagnstico.
Manual CTO de Medicina y Ciruga, 2. 8 edicin

La eva luacin de la extensi n tumora l se comp leta r med iante TC tora-


pTis lntratu bular coabdomina l. As se co nstat ar si la enfermedad est limitada al testculo
pT1 Testculo y epiddimo sin invas in vasc ular/ li nftica (estad io 1), o exist e afectaci n de ga ng lios infrad iafragmticos (estadio 11),
pT2 Testculo y epiddimo con invasin vascular/ linftica o t nica o bien si hay inclu so extensi n supra diafrag mti ca o a rganos slidos
vaginal (estad io 111). El sistema de estadificaci n utiliza m lt iples va ri aciones, pero
pT3 Afectacin de cord n espermti co q uiz la clasifi cac in ms aceptada sea la expuest a anteri ormente en la
pT4 Escroto Tabla 16.

N1 Gang lios menores de 2 cm


N2 Ganglios ent re 2-5 cm
N3 Gang lios mayores de 5 cm
7 .S. Diagnstico diferencial
M1 a Metstasis en ganglios no reg iona les o pulmn
M1 b Metstasis viscerales no pu lmonares Aunque frecuentement e la existencia de un t umor testicular no plantea
Sx Marcadores tu mora les sricos no disponibles dudas d iagnsticas, existen una serie de patologas testicu lares que, jun-
SO Nive les de ma rcadores normal es to con los tumores, pueden presenta rse en algn momento dent ro del
51 LDH < 1,5 x n ; y 13-HCG < 5.000; y AFP < 1.000 cuadro genrico q ue se deno mina "escroto ag udo", ca racterizado por el
52 LDH entre 1,5 x n y 1O x n o 13-HCG entre 5.000 y 50.000 aumento doloroso de volumen del contenido escrot al, acompaado o
o AFP ent re 1.000 y 10.000 no de sig nos infl amato ri os (Fig ura 24 y Tabla 18).
53 LDH > 1Ox n o 13-HCG > 50.000 o AFP > 10.000 Orquiepididimitis: suelen prese ntarse con do lo r intenso, enro-
j ecim iento cutneo, fi eb re, y a veces sinto m ato log a miccional.
n = lmite superi or norma l de LDH
En la explorac in , la elevacin del testcul o (signo de Prehn)
Agrupacin por estadios TMR testculo (abreviado)* d ism inuye el do lo r (s igno de Pre hn positivo). En su etiologa se
Estadio O pTis NO MO deben cons iderar grme nes d e tra n smisin sexua l en pacientes
Estadio 1 pT1 -4 NO MO adultos m enores de 35 a os, y u ropatge nos (E. coli) si superan
esta edad.
Estadio II A Cua lquier pT N1 MO
Torsin del cordn espermtico: suele apa recer en la infancia o
Estadio IIB Cualquier pT N2 MO la adolescencia, con dolo r de apa ricin bru sca y signos cutneos
Estadio IIC Cualquier pT N3 MO infla matorios crec ientes a medida que p rogresa el cuadro. El teste

Estad io 111 Cualquier pT Cualquier N M1 se encuentra horizo ntaliza do, y ocasionalm ente puede pa lparse la
espiral del co rd n t orsio nado. En este caso, la elevacin del t est culo
* No tiene en cuenta marcadores tumorales
incrementa la se nsaci n dolo ro sa.
Tabla 16. Estad ificacin clnica y patolgica Hidrocele y espermatocele: so n dos cua dros q ue raramente se
presentan de forma brusca y co n dolor ag udo, fc il mente d iferencia -
bies por la expl o racin y su transil um inacin posit iva y, ante la duda,
Tumor con una palabra aumenta 13-HCG med iante ecografa.
- Coriocarcinoma
Seminoma (en la mujer, el equivalente es el disgerm inoma)

Tumor con dos palabras aumenta a-fetoprotena


Carcinoma embrion ario
- Tumor del seno endodrmico

Tabla 17. Ma rcado res tumorales en t umores germ ina les (v lidos para
ova rio y testculo)

La vida media de la a-feto prot ena es de siete d as, frente a t res das de la
~- HCG Este dato es importa nte a la ho ra de va lorar la posible presencia
de enfermedad resi dual, ya que la elevaci n persistente de uno de estos
marcadores despus del t ratam iento supone la exist encia de tumor no
eliminado.

La ecog rafa testicular es un mt odo sencillo y fia bl e para la d iferencia-


cin entre m asas slidas y qusti cas, y su loca lizaci n exacta intratesticu-
lar o depend iente de los anejos. Cuando, a pesar de los marca dores, los
datos ecog rfi cos so n sugerentes de tum o r, est indica da la expl ora cin
qu irrg ica a t ravs de una incisi n ing uinal, para evita r la posibilidad te -
rica de im plantes t umorales en la piel escrota l y t ener mejor co ntro l de
pedcu lo vasc ulolinft ico a nive l del co rdn espermtico.

Si la exploracin co nfi rm a la presencia de una masa, el testculo debe ser


extirpado (orquiecto m a rad ical). Figura 24. Pieza macroscpica de t um o r t esticul ar

07 Tumores testiculares
Urologa 1 01
Orquitis Polaridad conservada ser una opcin teraputica. La q u imioterapia (BEP) es la nica
Prehn MEJORA posibilidad, presentando una tasa de curacin de alrededor del
el dolor 80%.
Torsin del cordn Testcu lo
espermtico horizontal izado Tumores no seminomatosos
Prehn EMPEORA
el dolor Estadio l. Existen tres posibil idades teraputicas t ras la orqu iecto-
Tumor testicular Masa palpable ma:
indolora Observacin y seguim iento peridico. Tasas de recadas de un
Tabla 18. Diagnstico diferencial de los tumores t esticula res 20%.
Linfadenectoma retroperitoneal de estadificacin, pues as se
tiene certeza del estadio y se reduce a un 10% las recidivas (en
7 .6. Tratamiento Europa no est extend ida esta prctica).
Quimioterapia profilctica (cisplatino), y as se reducen al 5% las
rec idivas.
Hasta la fecha no existe un protocolo nico de tratamiento, pudiendo
variar incluso de un centro a otro. Siempre se rea lizar orquiectoma En aquellos casos en que exista invasin vascular en la pieza de or-
radica l va inguinal. Poste riormente, el patlogo informar de la estir- quiectoma, la tasa de recidivas asciende aI SO%. Pa rece lgico, en es-
pe histopatolgica. A continuacin, el estud io de extensin mediante tos casos, inclinarse de entrada por una de las dos lti mas opciones.
TC toracoabdominoplvica y nuevos marcadores postorquiectoma. En La tasa de curacin alcanza el 98%.
funcin de la histologa y del estadio, se as ignar un tratamiento a cada Estadio lla-llb. Histricamente se realizaba linfadenectoma retro-
paciente. peritoneal completa como nico tratamiento, pero ante tasas de
recidiva no desdeables, actualmente se indica quimioterapia de
inicio (BEP). La tasa de supervivencia supera el 95%.
Estadio llc-111. Antes de la aparicin de la actua l quimioterapia, la su-
La orquiectoma en el cncer de testcu lo es va ingui- pervivencia era del 5-10%. Actua lmente, el esquema de tratamiento
nal. En el cncer de prstata, va escrota l. mayoritariamente aceptado es quimioterapia primaria.
Recuerda
El tratamiento de la neoplasia testic ular se resume en la Figura 25.
Semi noma

Se caracteriza por su gran radiosensibil idad, de ah que la radioterapia


Tumor testicular
haya sido la base del tratamiento de estos tumores. Actua lmente, la qui-
mioterapia obtiene resultados similares.
Estadio l. El tumor tericamente est lim itado al testculo. No obs- Orquiectoma radical
tante, se sabe que hasta un 20% de pacientes presentan micro-
metstasis a ganglios retroperitonea les, que en el momento del
diagnstico no se detectan . La presencia de micrometstasis se ha
relacionado con dos factores de riesgo: tumor de ms de 4 cm y la
t t t
lla, llb la lla,llb
invasin tumoral de la rete testis. En estos pacientes se aconseja ad-
ministrar rad ioterapia o quimioterapia (cisplatino). Con esto, frente a
Observacin
Si FR:
t Observacin
Linfadenectoma
+++ t
Radioterapi a inva sin Quimioterapia
los pacientes que optaron n icamente por observacin, la recidiva > 4 cm rete testis Qu imioterap ia (EEUU) vasc ular
Quim ioterap ia o
desciende del 20 al 5%.
(UE) linftica
Radioterapia
Quimi oterapia

El cisplatino produce vmitos con mucha frecuencia.


Otro efecto secunda rio es su nefrotoxicidad.
~
~
Seminoma/ No seminoma
IIC-111

Recuerda
Qu im ioterapia

Estadio lla-llb. En este caso, el tumor ya est extendido a gan-


glios retroperitoneales, y requiere, por tanto, tratamiento agresi- Figura 25. Algoritmo de tratamiento de la neoplasia testicular
vo. Se dispone de dos opciones: radioterapia sobre las cadenas
afectadas (teniendo en cuenta el teste afectado, se irrad iar a Masas residuales
unas cadenas ganglionares u otras), o qu im ioterapia BEP (c ispla-
tino, etopsido y bleom icina). Ambos obtienen resultados muy Se define como masa residua l la existencia de cong lomerados adenop-
similares. ticos tras tratam iento quim ioterpico o rad ioterpico. Cuando el tumor
Estadio llc-111 (estadios avanzados). El tumor tiene metstasis primario es un semi noma y existen masas residuales, la actuacin a segu ir
ganglionares retroper itoneales supe riores a 5 cm, o afectacin ser: si la masa residual es inferior a 3 cm, tiene muy pocas probabil i-
suprad iafragmtica o de vsceras s lidas. La radioterapia deja de dades de contener tumor residual y no requiere ms que observacin.
Manual CTO de Medicina y Ciruga, 2.8 edicin

Pero cuando es superior a 3 cm, se debe realizar una PET (tom ografa por
emisin de po sitrones), si existiera esta posibilidad a nivel tcn ico, pues
detecta con una alta sensibilidad y especificidad la presencia de tumor
residual. Si no se dispone de una PET o si esta es positiva, se rea liza r
ciruga de la masa.

< 3cm > 3cm


Cuando el tumor primario es un tumor no seminomatoso, se debe
t
realizar exresis de di cha masa siempre, con independencia del ta-
mao. !
Observacin ---e /
+/- PET

""'- Ea

La histologa de estas masas residuales, una vez extirpada s, puede ser:


t
Exresis de la masa residual
tejido necrtico hasta en un SOo/o de las ocasiones, tumor viable en un
15o/o y teratoma en un 35o/o (estos, dejados a libre evolucin, pueden con-
SO% necros is 35% te rato m a 15% tumor viable
vertirse en teratomas ma lignos o producir procesos compresivos con su
crecimiento) (Figura 26). Figura 26. Algoritmo de tratami ento de las masas residuales

" La cln ica ms frecuente es una masa escrotal indolora.


Ideas clave ~
" El seminoma nunca produce a-fetoprotena .
" El tumor testicular es la neoplasia slida ms frecuente en el va-
rn joven. " Como tratamiento, la orquiectoma por va inguinal se realiza en
todos los casos.
" La tasa de curacin es mayor del 90o/o.
" El semi noma es radiosensible. Por ello puede utilizarse radiote-
" El tumor testicu lar ms frecuente es el sem inoma. No obstante, rapia como tratam iento en los primeros estadios. Si se trata de
esto es muy variable segn la edad del paciente. un estadio avanzado, se empleara quimioterapia.

" Una masa testicu lar por encima de los 50 aos debe hacer pen- " Los tumores no seminomatosos se tratan con orquiectoma +
sar en un linfoma. quimioterapia. No obstante, si es un tumor limitado al testculo,
puede plantearse la vigilancia tras la orquiectoma.
" El tumor tpico de las disgenesias gonadales es el gonadoblastoma.

Qu diagnstico, entre los siguientes, es el ms probable en un


Casos clnicos 1 nio de 8 aos con signos inequvocos de pubertad precoz y que,
en la exploracin, presenta una masa en el testculo derecho de
Un hombre de 31 aos consulta por la presencia de una masa 2 cm de dimetro?
palpable en el teste derecho, de un mes de evolucin, no dolo-
rosa. Su urlogo le realiza una ecografa testicular, en la que se 1) Tumor de clulas de Leydig.
evidencia una lesin hipoecoica, bien delimitada, intratesticular. 2) Seminoma.
Los marcadores tumorales u-fetoprotena y P-HCG son negati- 3) Tumor del saco vitelina.
vos. La actitud ms correcta de, entre las siguientes, sera: 4) Teratoma .

1) Dado que los marcadores tumorales son negativos, se descarta RC:4


neoplasia testicular y requiere observacin .
2) Realizacin de una tomografa computarizada toracoabdmi-
noplvica.
3) Biopsia tran sescrotal del testculo.
4) Orquiectoma radical y esperar resultado del patlogo.

RC: 4

1) The most likely diagnosis is a pure semi noma.


Case Study . 2)
3)
The most likely diagnosis is a yolk sac tumor.
Tumor markers can be used for follow-up.
A 35-year-old mate presents to your outpatient clinic with a 4) The elevated 13-HCG is directly related to the presence of hema-
three-month history of painless swollen testicle. Tumor markers togenous metastatic lesions.
u-FP y p-HCG are elevated. Which of the following is the most
appropriate answer about this case? Correct answer: 3

07 Tumores tes t iculares


Urologa 1 01
A 32-year-old patient presents to his physician saying that his 3) Hydrocele.
testicles have increased in size for the last two months. Physical 4) Testicular torsion.
examination reveals a palpable lump in his right testicle. Ultra-
sound shows an intraparenchymatous lesion with calcifications. Correct answer: 2
Mark the correct answer:
A 32-year-old patient reports to the physician a three-week his-
1) An inguinal orchiectomy is indicated. tory of palpable lump in his testicle. Physical examination re-
2) A transscrotal orchiectomy is indicated. veals increased size of the right testicle with a painless petrous
3) A contra lateral testicle biopsy should be performed. nodule. Testicular echography reveals an intraparenchymatous
4) The patient should receive neoadjuvant chemotherapy. lesion that meas u res 2 cm and has no echogenicity. Which of the
following tests or maneuvers would not be indicated in this pa-
Correct answer: 1 tient?

Which of the following options is the most likely diagnosis of 1) Thoraco-abdomino-pelvic CT sean in order to stage the disease.
a patient presenting with a scrotal mass that feels like a bag of 2) A blood sample in order to determine f3-HCG levels.
worms? 3) Schedule a radical orchiectomy as soon as possible.
4) Perform a bilateral testicular biopsy.
1) Pure semi noma.
2) Varicocele. Correct answer: 4
.---------UroloW-a_

Trasplante renal

'
Lo ms importante de este tema
8.2. Contraindicaciones
ORIENTACIN
son los tipos de rechazo. Se debe
ENARM insistir en el rechazo agudo.
Las contraindicaciones absolutas son las sigu ientes:
Infeccin activa.
Enfermedad maligna que no pueda ser erradicada.
Sospecha de no cumplimi ento teraputico del protocolo inmu no-
8.1. Indicaciones supresor.
Glomerulonefritis activa.
Expectativa de vida reducida por enfermedad de base del paciente.
Las dos enfermedades que ms comnmente abocan a una insuficiencia Presencia de anticuerpos preformados frente a antgenos del donante.
rena l termina l irreversible, tratable mediante un trasplante rena l, son la
g lomerulon efritis y la diabetes mellitus insulinodepend iente. En referenc ia a las contraindicaciones relativa s del trasplante renal se
debe decir que estas se han ido modificando a lo largo de los aos, al
Otras causas im portantes son: mejorar la tcn ica y los cu idados prequirrgicos y posquirrgicos. En mu-
Pol iquistosis renal. chas ocasiones, el trasplante plantea menos riesgo que una hemodilisis
Nefroesclerosis hipertensiva. crnica.
Enfermedad de Al port.
Nefropata lgA. Actua lmente se cons idera n cont raind icaciones re lativas la edad avanza-
Lupus eritematoso sistm ico. da, la oxalosis, la am iloidosis, la enfermedad iliofemora l oclusiva, las ano-
Nefroesclerosis. ma las del tracto urinario inferior o las alteraciones psiqu itricas graves.
Nefritis intersticial.
Pielonefritis.
Uropata obstructiva. 8.3. Complicaciones
Los mejores receptores son individuos jvenes cuyo fa llo renal no se deba a
una enfermedad sistmica que pueda daar el rin trasplantado o causar la Las comp licaciones que se pueden presentar son las siguientes:
muerte por causas extrarrenales. Generalmente se suele mantener al recep- Rechazo (Tabla 19).
tor en tratamiento con dilisis durante un cierto tiempo previo al trasplante. Recurrencia de la enfermedad en el rin trasplantado.

Patogenia Tratamiento
Hiperagudo Minutos, das Ac. preformados Trombosis microvascular Nefrectoma del injerto
CID Isquemia o infarto
Act. del complemento PMN en capila res
Dao endoletelial
Acelerado Das Celular (+Ac) Vascul itis necrotizante Bolos de estero id es
Respuesta 2.ia a Ag-HLA Ac monoclonales
Agudo Semanas Celular (+Ac) Forma vascular: mediada por Ac Bolos de esteroides (la vascu lar suele ser resistente)
Infiltrado de linfocitos Forma celular: tubulointersticial Ac monoclonales
Crnico Meses, aos Humoral y celular fntima arterial aumentada No hay; control de HTA
Atrofia tubular
Glomeru lopata
Tabla 19. Rechazo en el trasplante renal
Urologa 1 08
Complicaciones tcnicas. Compli caciones vasculares, hemorra- ciones gastrointestinales, hiperparatiroidismo y tumores (cncer de
gia, hipertensin por estenosis de la arteria rena l, trombosis ve- piel y de labios, carcinoma in situ de crvix, li nfomas no Hodgkin;
nosa, comp licaciones del tracto urinario, necrosis tubu lar aguda, guardan re lacin con el tratam iento inmunosupresor). Puede apa-
linfoceles. recer hipertensin debida a enfermedad en los riones originales,
Complicaciones no tcnicas. Infecciones bacterianas y oportun is- como consecuencia de rechazo, por estenosis de la anastomosis de
tas en relacin con la inmunosupresin, hiperglucemias, complica- la arteria renal o por toxicidad renal por ciclosporina.

" La clnica caracterstica del rechazo agudo es: fiebre, hiperten-


Ideas clave ~ sin y dolor en el rea del injerto.

" Las causas ms frecuentes de insuficiencia renal crnica son la " El rechazo agudo produce oliguria, no poliuria.
diabetes mellitus y las glomerulonefritis.

presenta fiebre de 38 oc,


TA de 180/11 O, oliguria y disminucin
Casos clnicos en la concentracin urinaria de sodio. El diagnstico ms pro-
bable sera:
Un paciente de 35 aos, con insuficiencia renal crnica, secun-
daria a pielonefritis crnica recibe un trasplante renal de cad- 1) Infeccin respiratoria.
ver con el que comparta dos identidades en A y By una en DR. 2) Pielonefritis aguda del injerto renal.
Recibe tratami ento inmunosupresor con ciclosporina A y cor- 3) Recidiva de su enfermedad renal.
ticoides en dosis estndar. En el posoperatorio inmediato se 4) Rechazo agudo del injerto renal.
observa buena diuresis, y no es necesario el tratamiento susti-
tutivo con hemodilisis. En el 5. 0 da de evolucin, el paciente RC: 4

Urologa_

Uropata obstructiva

Tiempo de evolucin. La ag uda suele cursa r con dolo r (clico nefr-


OR I ENTACIN Se debe tener una idea general tico), siendo la crnica m s frecuentemente asintomtica .
y aprenderse bien las Ideas
ENARM Clave.
Lugar de obstruccin:
Tracto urinario inferior (uretra y vejiga). Cursa co n retraso
para iniciar la micc in, d ism inuci n de fu erza y del tamao del
chorro, got eo te rmin al. hematu ria, escozor al orinar, orina turbi a,
retencin aguda de orina o incontinencia pa radjica ("miccin
9.1. Caractersticas por rebosamiento").
Tracto urinario superior (urter y rin). Estos pacientes pre-
sentan dolor en el flanco (rin y urter proximal), dolor en fla n-
Detencin del flujo de orina en cualquier punto entre los c lices re- co con irradiacin a genitales (u rter medio) o sndrome miccio-
na les y el exterior (Tabla 20) . nal (urter terminal).
Su importancia reside en el desarrollo potencial de insuficiencia re-
nal, por lo que ti ene importan cia la obstruccin urinaria bilateral o la Despus de resolverse una obstru ccin, sobre todo si es crnica, puede
unilatera l sobre rin nico funcionante. producirse una fase de poliuria.
Una obstru cc in de ms de un mes de duracin puede dar lugar
a un dao rena l funcional y estruct ural permanente. Esto se debe a que, a nive l tubular, cuando se ralentiza crnicamente el
flujo urinario, se genera una insensibilidad a la ADH transitoria (diabetes
inspida nefrognica), de ah la po liuria.

9.3. Diagnstico

El d iag nsti co de seg uridad, la va lo ra ci n de la evolucin, y el pronstico


9.2. Clnica son ecog rfi cos.

Ade ms, son tiles la anamnesis y la exp loraci n fsica, la radiologa


La forma de presentacin depende de los sig uientes factores: simp le (convenie nte en litias is rad ioopaca), urografa intravenosa (con-
Etiologa de la obstruccin. Presenta la clnica propia de la enfer- firma una posible anomal a func io nal y til en litiasis radiotransparen-
medad de base. te), cistografa, estudio metablico (til en prevencin de recid ivas), TC

lntraluminal lntraparietal Extra parietal (compresin extrnseca) Extraparietal (disfuncin neuromuscular)

Litiasis Estenosis congnita Urter retrocavo Vejiga neurgena


Tumores (hipernefroma, Estenosis postinfecciosa Rin en herradura Vejiga automtica: lesin sobre L1-L3
urotelioma) Estenosis postraumtica Fibrosis retroperitoneal Vejiga tona: lesin bajo L1-L3
Necrosis papilar Estenosis isqumica Tumores (prstata, vejiga, ginecolgicos) Disfuncin de la unin pieloureteral
Cogulos Hiperplasia prosttica Reflujo vesicoureteral
Ligadura iatrgena de urteres

Tabla 20. Clasificac in de la uropata obstru cti va


Urologa 1 09
abdominal, ecog rafa t ra nsrectal, bio psia prost tica d irig ida, pielografa Manejo de la uropatia obstructiva
retrgrada, nefrostog rafa, cistoscopia, fluj ometra, cistomanometra y
citolog a urin aria.
lnfravesical Supravesical
(globo vesical) (no globo ves ica l)

9.4. Tratamiento t
Ecografa renal

Es necesario resta blecer el fl uj o urinari o. La mayor parte de las veces se


rea liza mediante litotricia o correccin qui rrg ica.
t
No dil ataci n Dil atacin unilatera l
t
Dilatacin

t ~
bilateral

Si la obstru ccin es ag uda y/o bilateral, la desobstruccin es urgente, y

j
Estudio mdico Birreno Monorreno

~
puede logra rse mediante un sondaje vesica l, ta lla vesica l, catter uretera l
o nefrostom a.
Estudio (U IV, TC)
Fracaso rena l

En caso contrari o, hay q ue va lo rar el g rado de suf rimi ento rena l y su


revers ib ilid ad . En los casos en qu e hay destru cc in irreve rsib le de la tL _ Deri vac~n de via
urinaria superior
va urin aria, es necesa ri o rea liza r un a derivac i n urinaria definit iva
(nefrostoma/doble J)
(Figura 27).
Figu ra 27. Procedimiento de act uacin frente a la uropat a obstructiva

" El dolor sue le estar presente en la obstruccin aguda . Sin em-


Ideas clave RS bargo, en la crnica, es frecuente su ausencia.

" La uropata obstructiva puede producir insuficiencia renal si no " Despus de resolver una uropata obstructiva puede producirse
se resuelve a tiempo. una fase de poliuria.

eaS e St udy ! ~' ~


1)
2)
3)
A suprapubic cystostomy shoul d be performed .
Foley cat heterization should be tried.
Anticholinerg ic agent s are ind icated .
A 78-year-old patient is brought to the emergency department 4) Pelvic floor muse/e trai ning exercises are recommended.
with complaints of urinary incontinence. On the one hand, he
says he is no trouble urinating when he wants to and on the other Correct answer: 2
hand he says he has constant urinary leakages all day long. A few
years ago he felt similar symptoms and had a notable improve-
ment after initiating treatment with finasteride. In this case:
Uro logia_

Disfuncin erctil

Enfermedad cardiovascular: cardiopata, hipertensin arterial, en-


ORIENTACIN Se debe incidir en los factores
de riesgo, el tratamiento fermedad vascu lar perifrica y descenso del colesterol HDL se han
ENARM y sus contraindicaciones. relacionado de manera clara con la disfuncin erctil.
Tabaquismo: factor independiente.
Secundaria a frmacos: aquellos que causan hiperprolactinemia, que
disminuyen los niveles de testosterona, psicotropos y antihipertensivos.
10.1. Introduccin Secundaria a consumo de drogas: cocana, herona, etctera.
Trastornos afectivos: depresin.

La disfuncin erctil (DE) se defin e como la inca pacidad persistente o re-


cu rrente para conseg uir o mantener la suficiente rigid ez del pene que 10.5. Diagnstico
permita una relacin sexual satisfactoria. Debe tener una duracin mni-
ma de tres meses.
El diagnstico debe basarse en los siguientes componentes:
Historia clnica y sexual: investigar los posibles factores de riesgo
10.2. Prevalencia implicados.
Exploracin fsica: encaminada a descartar enfermedad vascular,
enfermedades neurolgicas, trastornos genitales y endocrinopatas.
En Estados Un idos, en un estud io en varones de entre 40 y 70 aos, se En varones mayores de 50 aos se incluir tacto rectal.
estim que la prevalencia g lobal era del 52%. Determinaciones analticas: g lucemia basa l. perfil lipdico, testos-
terona total y libre y prolactina. Adems, es conveniente solicitar he-
mograma, func in renal y heptica.
1 0.3. Etiologa Pruebas especializadas: nicamente en ocasiones muy seleccionadas.

Se puede clasificar en: 1 0.6. Tratamiento


Orgnica. Causas vasc ulares (las ms frecu entes 60-80%), neurolgi-
cas (1 0-20%), hormonales (5-1 0%) o locales.
Psicgena. Se puede estructurar en tres escalones o etapas, que sern superadas
Mixta. En la mayora de los casos de etiologa orgnica se aade un en funcin de fracaso del escaln previo.
compo nente psicolgico.
Frmacos orales

1 0.4. Factores de riesgo lnhibidores de la fosfodiesterasa S (sildenfafi lo, vardenafilo, tada-


lafilo): son el tratamiento de eleccin. Induce la relajacin de mscu-
lo liso del cuerpo cavernoso, liberando xido ntrico (NO). El NO es el
Edad: factor independiente. principal neurotransmisor de la ereccin. Precisa de deseo sexual y
Diabetes: es la enfermedad endocrina ms frecuente asociada a dis- estimulacin previa para que tenga efecto.
funcin erctil. Sign ifica una probabilidad tres veces superior de pre- Las contraindicaciones absolutas de sildenafilo son:
sent ar DE. Estn implicados meca nismos vascula res, neuropticos y Administracin concomitante con nitratos o frmacos donado-
disfuncin gonadal. res de xido ntrico por el riesgo de hipotensin grave (dinitra-
Urologa 1 10
to/monon itrato de isosorbida, molsidom ina, nicorandil, nitrog li- Segunda lnea
cerina, nitroprusiato sdico).
Pacientes en los que est desaconsej ada la actividad sexua l Terapia intracavernosa: alprostadil (PGE1 ), mediante inyeccin di-
(ang ina inesta ble, insu fi ciencia ca rdaca o infarto reciente, hace recta en los cuerpos cavernosos. Otros frmacos son la papaverina y
menos de seis meses). la fento lam ina.

Apomorfina: agonista dopaminrgico que acta a nivel centra l so- Tercera lnea
bre el mecan ismo de la ereccin.
Est contraind icado en sujetos que tengan desaconsejada la activi- Ciruga de revascularizacin (venosa, arterial). Impla nte de pr-
dad sexual. tesis de pene.

" El sildenafilo est contraindicado en pacientes que t oman ni-


Ideas clave ~ tratos o frmacos donadores de xido ntrico, en pacientes con
infarto agudo de miocardio (IAM) en los ltimos seis meses, y en
" La causa ms frecuente de disfuncin erctil es vascul ar. pacientes con insuficiencia cardaca grave o angina inestable.

" La enfermedad endocrina ms relacionada con ella es la diabe-


tes mellitus.

1) Verapamilo.
Casos clnicos 2) Digoxina .
3) lndapam ida.
Paciente de 63 aos, en tratamiento a demanda con citrato de 4) Mononitrato de isosorbida .
sildenafilo por presentar disfuncin erctil de aos de evol ucin.
Seale cul de los siguientes frmacos NO asociara en ningn RC:4
caso a su tratamiento:

A 78-year-old male goes to you r outpatient clinic complaining


Case Study . _ of erecti le dysfunction. He has never taken any specific medical
treatment. Mark th e incorrect statement:
A 65-yea r-old patient, with a history of hypertension, goes to
his physician asking for a diagnostic study for his erectile dys- 1) Most patients with erectile dysfunction will be treated w ith
function . Which of the following options would not be included non-specific treatment or t hera pies.
in the initial diagnostic workup? 2) The basic diagnostic workup of such a patient should incl ude
the identification of revers ible risk factors.
1) Sexual history. 3) Lifestyle changes and controlling risk fa ctors have no role in the
2) Digital rectal examination. current treatment of erectile dysfunction.
3) EKG. 4) The early use of an elevated dose of sildenafil after a radical
4) Measuring his total and free testosterone levels. prostatectomy is associated with preservation of smooth mus-
ele fibers in the human cavernous bodies.
Correct answer: 3
Correct answer: 3
1
Urologia_

Traumati smos del aparato


genitouri nario

o 11 1 1 r,J 1 11 e 1 u r~
Se han de repasar bien pable secunda ria a hematoma retroperitoneal o a urinoma y disten-

ENARM las Ideas clave. sin abdominal.

Las complicaciones inmediatas ms frecuentes son : hemorragia, ex-


travasacin urinaria que secundariamente da lugar a un absceso y
Las lesiones del aparato genitourina rio son frecuentes en los pacientes septicem ia.
politraumatizados, as que, ante todo paciente con fracturas costa les ba-
jas, equimosis o masa en fl ancos, fractura s de las apfisis t ransversas, frac- Las comp licaciones ta rdas ms importantes son: hipertensin, hidrone-
turas de los cuerpos vertebrales y/o fracturas plvicas, debe sospecharse frosis, fstu la arteriovenosa, formacin de c lculos, y pielonefritis y la he-
una lesin de este t ipo. morrag ia tarda.

Diagnstico
11.1. Lesiones del rin
Para determinar el g rado de les in re nal y su funcin, es fundamental
realizar una urografa intravenosa (UIV) que establezca la presencia
Son las lesiones ms frecuentes del aparato urinario, siendo el mecanis- o ausencia de ambos riones, defin a con claridad los contornos re-
mo ms frecuente (80-85%) el traumatismo contu so directo en el abdo- nales, los lm ites cortica les y delimite los sistemas colectores y los
men, fl anco o reg in dorsa l. urteres.

Clasificacin patolgica En el paciente politraumatizado, puede aportar mayor informacin la rea-


lizacin de una TC.
Traumatismo renal menor (85%): eng loba los grados 1y 11 de la
asociacin americana pa ra la ciruga del trauma (AAST). Incluyen la Si no se determina en su totalidad la extensin de la lesin, puede aso-
contusin del parnquima (los ms frecuentes), el hematoma cap- ciarse una nefrotomografa o una TC, que adems muestra el estado de
su lar y las laceraciones corticales superficia les. Rara vez requieren los rganos vecinos.
exploracin quirrgica .
Traumatismo renal mayor (15%): grados 111, IV y V de la AAST. La arteriografa se indica cuando el rin no se observa bien en la uro-
Laceraciones co rt icomedulares profundas que p ueden afectar al grafa excretora.
sistema colector, con extravasacin de o ri na al espacio perirrena l.
Se acompaa a menudo de hematomas retrope ritonea les y peri- La s causas ms importantes que no permiten la adecuada observacin
rrenales. en la TC o UIV son : rotura total del pedculo, trombosis arterial, contu-
Lesin vascular (1% de los tra umatismos contusos): grado V. sin intensa que causa espasmo vascular y la ausencia de ri n.

Clnica Los ex menes con istopos, en la evaluacin de urgencia, son menos


sensibles que la arteriografa o la TC.
La hematuria macroscpica o microscpica despus de un traumatismo
indica lesin del aparato urinario, aunque no apa rece en todos los casos Tratamiento
(por ejemplo, ante lesiones del pedculo vascu lar). El grado de hematuria
no siempre se corresponde con el grado de la lesin. Las medidas teraputicas iniciales deben dirig irse a la estabilizacin he-
modinmica y reanimacin completa del paciente, con tratamiento del
Otros sntomas y signos son el dolor abdominal o en un flanco, equi- shock, control de la hemorragia y la evaluacin de la s lesiones concu-
mosis en los flancos o cuadrantes su perio res de l abdomen, masa pal- rrentes.
Urologa 1 11
El tratamiento quirrgico est indicado en: La rotura puede ser:
Todo paciente inestable (hemorragia retroperitoneal, lesin del pe- Extraperitoneal: perforaciones por fragmentos de fracturas plvi-
dculo renal, extravasacin urinaria). cas. Son las ms frecuentes.
Los traumatismos renales por penetracin (salvo si se ha podido de- lntraperitoneal: golpes directos cuando la vej iga est llena.
term inar el grado de la lesin y resu lta ser una les in menor del pa-
rnqu ima sin extravasacin urinaria). Clnicamente pueden manifestarse con dificultad para la miccin, hema-
En el tratamiento de las complicaciones como el urinoma retroperi- turia macroscpica, dolor plvico o en hemiabdomen inferior, abdomen
toneal o el absceso perirrenal, la hipertensin maligna que requiere agudo (indica rotura intraperitoneal). Como complicacin tarda puede
reparacin vascular o nefrectoma, y en algunos casos de hidrone- aparecer una incontinencia parcial en las lesiones que se extienden hasta
frosis. el cuello vesical.

Una cistografa demostrar una rotura vesical. La cistoscopia no est in-


11.2. Lesiones del urter dicada porque la hemorragia y los cogulos impiden una buena visuali-
zacin.

Son raras, pero pueden ocurrir durante el cu rso de intervenciones qu i- Tratamiento


rrgicas plvicas, por heridas de ba las, desaceleraciones rpidas en ac-
cidentes, en manipulaciones endoscpicas de clcu los o en resecciones Las roturas extraperitonea les se tratan con cateterizacin uretral perma-
transuretrales. nente y cistostoma suprapbica. Slo si persiste la extravasacin, es ne-
cesario el tratamiento qu irrgico.
La ligadura del urter conduce a la aparicin de hidronefrosis con fie-
bre, dolor en flanco, nuseas, vmitos e leo, y si es bilateral, anuria. Las roturas intraperitoneales deben repararse por va transperitoneal (ci-
ruga reparadora), debiendo dejar cistostoma suprapbica.
Si lo que ocurre es extravasacin, se forma un urinoma que secunda-
riamente provoca estenosis y fibrosis reactiva, junto con hidronefrosis;
si se extravasa hacia la cavidad peritoneal, da lugar a una peritonitis 11.4. Lesiones de la uretra
aguda.

En las pruebas de laboratorio, si hay una lesin por t raumatismo externo, Son poco frecuentes, ms habitua les en varones y generalmente aso-
aparece hematuria microscpica en el 90% de los casos. ciadas a fracturas plvicas y a contusiones directas. Se considera ure-
tra posterior la porcin proximal al diafragma urogenital y anterior, la
El diagnstico se realiza mediante una UIV o uretrografa retrgrada. En el distal.
perodo postoperatorio inmed iato, la urografa es el mejor mtodo para
descartar la lesin ureteral. Lesiones de la uretra posterior
(prosttica y membranosa)
Tratamiento
La primera medida es la derivacin urinaria mediante nefrostoma. Re- La uretra membranosa se lesiona con ms frecuencia. Los pacientes
quieren tratamiento quirrgico inmediato. aquejan dolor abdom inal bajo e incapacidad para la miccin. El signo
Lesin del tercio inferior del urter: el procedimiento de elec- ms importante es la presencia de sangre en el meato de la uretra (ure-
cin es la reimplantacin en la vejiga. La ureteroureterostoma trorragia). En un tacto rectal, puede revelarse la presencia de hematoma
primaria puede indicarse si hay un corte transversal del urter. Se plvico y desplazam iento de la prstata hacia arriba.
usa transureteroureterostoma, si hay urinoma extenso e infec-
cin plvica. La prueba diagnstica ms importante es la uretrografa. La cateteriza-
Lesin del tercio medio y superior: ureteroureterostoma primaria cin o ureteroscopia no deben realizarse porque conllevan un alto riesgo
o sustitucin ureteral. de producir hematoma e infeccin y dao ms amplio de los desgarros
parciales de la uretra.
Es frecuente dejar un catter de doble-J transanastomtico, que se
retira despus de tres a cuatro semanas de cicatrizacin, con obje- Entre las complicaciones destacan estenosis, im potencia e incontinencia.
to de conservar el urter en una posicin adecuada con un ca li bre
constante, impedir la extravasacin urinaria y conservar la desviacin Su tratamiento es la cistostoma, seguida de ciruga diferida.
urinaria.
Lesiones de la uretra anterior
(pendular y bulbar)
11.3. Lesiones de la vejiga
Generalmente hay antecedentes de cada o maniobras con instru-
Frecuentemente se presentan debidas a fuerzas externas y asociadas a mentacin. Se manifiestan con hemorragia y dolor en el perin, pu-
fracturas plvicas. La lesin iatrgena ocurre en cirugas plvicas, gineco- d iendo existir infeccin por extravasacin y estenosis tarda. No debe
lgicas, herniorrafias y en intervenciones transuretrales. intentarse pasar catter uretral y debe evitarse la miccin hasta des-
Manual CTO de Medicina y Ciruga , 2.a edicin

cartar la existencia de extravasaci n. Se d iagnost ican med iante uretra- Sospecha de traumatismo urolgico
grafa retrgrada.
t
Tracto inferior Tracto superior
Tratamiento (uretrorragia) (hemat uria)

Si existe laceracin, debe rea lizarse ci stostom a suprapbica.


t t
Uretrografa UIVoTC

11.5. Lesiones del pene ~Lesin


Normal
~
Funcional
No funci onal

Evaluar
Sondaje vesical Cistotoma Arteriografa
Du rante las relaciones sexuales, puede ocurrir rotura de la tnica albug- segn grado
nea (fractura de pene). Provoca dolor y hematoma y precisa tratamiento t
Sospecha lesin vesical
qui rrgico.
(si hay hematuria)

11.6. Lesiones de los testculos Cistog rafa (+250 mi)

t
Rotu ra
Rotura
Para defin ir el da o, se rea liza ultrasonografa. Si hay rotura, se trata quirr- Extraperi tonea l
lnt raperitonea l
(lo ms frecuente)
gica mente. En el resto de los casos, el tratamiento es conservador.
t
Ciruga So nd aj e
A modo de resumen de todo lo anteri orm ente expuesto, la Figura 28
reparadora
recoge los procedimientos generales de actu acin frente a los t rauma-
tismos urolg icos. Figura 28. Ma nejo de los t raumatismos urolg icos

" Requieren tratamiento quirrgico los pacientes inestables, los


Ideas clave J!iS tra umat ismos renales por penetracin con extravasacin urina-
ria, urinomas retroperitonea les o abscesos perirrena les.
" La hematuria macroscpica o microscpica despus de un trau-
matismo indica lesin del aparato urinario. " Las lesiones vesica les pueden ser extraperitonea les, que al igual
que las lesiones de uretra, se asocian a fracturas plvicas, e intra-
" Pa ra determinar el grado de les in renal y su funcin, se puede peritoneales, producidas por golpes directos cuando la vejiga
realizar una urografa. En el paciente politraumatizado, puede est llena.
ser ms til la TC.

2) Las secuelas pueden ser graves y deben preveni rse con ciruga
Casos clnicos precoz.
3) En muchos casos se res uelve con compresin de la zona afecta.
Paciente que, tras una cada a horcajadas, refiere sangrado po r el 4) Lo ms probable es que precise un ingreso hospitalario prolon-
meato uretral con d ificu ltad para la miccin. Presenta un hema- gado.
toma perineal importante.
RC: 3
1) Debe real izarse sondaje inmed iato para preven ir compl icacio-
nes. Paciente precipitado desde 4 m de altura mientras limpiaba
2) Habr que real izar una TAC abdomina l para descartar les iones unos cristales. Tras valoracin y estabilizacin iniciales, se deci-
asociadas, como en cu alq uier lesin de uretra posterior. de su traslado, durante el que el paciente presenta una miccin
3) Debe rea lizarse reconstruccin quirrgica inmediata en caso de hem atrica.
una rot ura completa.
4) Debe realizarse una uretrografa retrgrada para valorar la gra- 1) Se debe probablemente a una contus in ves ical y no debe dr-
vedad de la lesin. sele mayor importancia.
2) Puede deberse a lesiones a varios niveles del aparato urinario y
RC:4 una TAC abdominal ser til para identificarla.
3) Esto permite focaliza r la lesin, descartando as otras lesio nes
Varn de 15 aos que p resenta mareo y palidez tras la visin de abdom inales.
sangre que parece provenir de los genitales. Niega t raumat ismo 4) No se debe co locar una sonda vesica l porque no ha presentado
alguno, aunque admite que se masturba ba cuando sucedi. uretrorrag ia previa a la m iccin.

1) Lo ms probable es que se trate de una rotura parcia l de uretra RC: 2


anterior.

11 Traumatismos
del aparato genitourinario
_UrololJ-L""'._____ __

Recomm ended reading 1

An abdominal ultrasound is performed on a 42-year-old patient for


another reason. During the study, an image such as the one shown
below is observed in the kidney. lndicate which do yo u consider to be
the most probable diagnosis [Figure la]:

1. Hydronephrosis.
2. Hypernephroma.
3. Simple renal cyst.
4. Renal abscess.
S. Angiomyolipoma.

When complementary abdominal examinations are performed, pa rticu-


larly ultrasounds, it is common for renal masses to be fou nd incidentally,
such that, currently, the number of renal carcinomas detected in a casual
manner exceeds the number of symptomatic renal carcinomas. However,
in this case it is not a hypernephroma, but a completely ben ign condi-
tion: a simple renal cyst (we may observe t wo of them in the ultrasound Figure la.
shown).
of age. In order to make a definitive diagnosis, the fo llow ing criteria must
An abdominal ultrasound makes it possible to distinguish a solid mass bemet:
from a cystic mass. Th is feature is essential for the study of renal masses, Imperceptible wall in the ultrasound.
since most sol id masses correspond to adenocarcinomas, whereas cystic That the limits thereof are well-detined.
masses are almost always simple renal cysts. Water densit y, w ithout intern a! echoes.
Presence of posterior reinforcement in the ultrasound.
A simple renal cyst is a benign, non-neoplastic mass that contains fluid. In the event that a CT or an intravenous urography is performed,
lt is very frequent, since it affects over half the population over 55 years there should be no evidence of contrast being captured.
Urologa

Recommended reading 2

A 35-year-old man visits the physician beca use he has palpated a mass
in his left testicle. He has no h istory of t rauma or high-risk sexual rela-
ti ons. The examination confirm s the presence of a mass and t he atta-
ched ultrasound is performed . W hat is the most adequate approach?
[Figure 2a]:

l. FNAC of the mass.


2. Surgical biopsy ofthe mass.
3. Inguinal orchiectomy, following collection of blood sample t o de-
te rm ine AFP and b-hCG.
4. Transscrot al orchiect omy with hemiscrot al resection.
S. Monitoring by ultrasound, since it is a normal testicula r image.

Testicular tumours are the most frequent mal igna nt neoplasias in ma les Figure 2a.
between 20 and 35 yea rs of age, except for leukaem ias. The most fre-
quent form of cli nica l presentation is as a scrot al mass. before and after the orchiectomy we should measure the levels of hCG and
AFP High concentrations of hCG and AFP decrease according to first-order
A testicular ultrasou nd is a simple, reliable method used to distingu ish bet- kinetics; the half-life of hCG is 24-36 hours and t hat of AFP is 5-7 days. These
ween solid and cystic masses. lt is also very useful to determ ine whether two markers should be serially analysed during and after the treatment.lf any
a mass is dependent on the test icle or the testicu lar annexes, as well as its or both of these markers increase again or do not decrease according to the
intratesticular position. In the ultrasound shown, we observe a very intense expected half-life, we should suspect tumour persistence or recurrence.
destructuration of the testicular parenchyma, which has a clearly heteroge-
neous appearance. Below, we show the appearance of a norma l testicu lar Fuente: Garca Macarrn J. Casos clnicos en imgenes. Madrid. CTO Edi-
ultrasound, such that you may appreciate the differences. torial, 20 12.

When, after th e clinica l examination


and the ultrasound, the d iagnosis is
still not clear, a t ransingu ina l surgica l
examination shou ld be performed. lf
this examination confirms the presen-
ce of a mass, the testicle should be re-
moved (answer no. 3 correct).

Overall, 70% of testicu lar tumours ma-


nifest sorne marker. lt is important to
determine these markers, not only for
diagnostic purposes, but primarily w it h a
view to the subsequent follow-up, since Figura 2b-2c. To the right, norm al testicle, the parenchyma whereof is visua lised as homogeneous in
an increase in them cou ld warn us early the ultrasound. On the contrary, the left image shows a destructured parenchyma, w ith heterogeneous
on about a recurrence. For this reason, echogen icity and sorne ca lcifi cation, wh ich suggests ma lignancy.
UrO 1Og a 1 Bibliografa

Bibliografa
Urologa

DiJ Castieiras Fernndez J. Libro del Residente de Urologa. Grficas DiJ Jimnez Cruz JF, Rioja Sanz LA. Tratado de urologa. Prous Science,
Marte, Madrid. 2006. Barcelona. 2006.

DiJ European Association of Urology. Guas clnicas Europeas 201 O. DiJ Resel Estvez L, Moreno Sierra J. Tratado de oncologa urolgica.
EAU, Arnhem. 201 O. Grupo Saned, Madrid, 2003.

DiJ Grupo CTO, Manual ao de Urologa. 9. ed., CTO Editorial, Madrid, DiJ Wein AJ, Kavoussi LR, Novick AC, Partin AW, Peters CA. Campeii-
2014. Walsh Urologa. Panamericana, Argentina. 2008.

También podría gustarte